Bac – Spécialité mathématiques – Métropole – sujet 1 – 11 septembre 2023

Métropole – 11 septembre 2023

Spécialité maths – Sujet 1 – Correction

L’énoncé de ce sujet de bac est disponible ici.

Ex 1

Exercice 1

  1. Pour tout réel $x$ on a :
    $\begin{align*} f(x)&=x\e^{x^2-3} \\
    &=\dfrac{1}{2}\times 2x\e^{x^2-3} \end{align*}$
    Ainsi $f(x)$ est de la forme $\dfrac{1}{2}u'(x)\e^{u(x)}$ où $u(x)=x^2-3$.
    Une primitive de la fonction $f$ est donc la fonction $F$ définie sur $\R$ par $F(x)=\dfrac{1}{2}\e^{x^2-3}$.
    Réponse d
    $\quad$
  2. Pour tout $n\in \N$ on a
    $\begin{align*} u_{n+1}&=\e^{2(n+1)+1} \\
    &=\e^{2n+2+1} \\
    &=\e^2\e^{2n+1} \\
    &=\e^2u_n\end{align*}$
    $\left(u_n\right)$ est une suite géométrique de raison $\e^2$.
    Réponse c
    $\quad$
  3. On doit écrire $\text{u <= 10000}$.
    Réponse a
    $\quad$
  4. Pour tout $n\in \N$ on a :
    $\begin{align*} v_{n+1}&=u_{n+1}+60 \\
    &=1,2u_n+12+60 \\
    &=1,2u_n+72 \\
    &=1,2\left(u_n+60\right) \\
    &=1,2v_n\end{align*}$
    La suite $\left(v_n\right)$ est géométrique de raison $1,2$.
    Réponse b
    $\quad$

 

Ex 2

Exercice 2

  1. a. On a $\vect{AB}\begin{pmatrix}2\\-1\\3\end{pmatrix}$ et $\vect{AC}\begin{pmatrix}1\\-2\\0\end{pmatrix}$.
    Or $\dfrac{1}{2}\neq \dfrac{0}{3}$. Ces deux vecteurs ne sont pas colinéaires.
    Les points $A$, $B$ et $C$ définissent bien un plan.
    $\quad$
    b. $\vect{CD}\begin{pmatrix} 2\\1\\-1\end{pmatrix}$
    D’une part $\vect{CD}.\vect{AB}=4-1-3=0$.
    D’autre part $\vect{CD}.\vect{AC}=2-2+0=0$.
    $\vect{CD}$ est orthogonal à deux vecteurs non colinéaires du plan $\mathscr{P}$. Il est donc orthogonal à ce plan.
    La droite $(CD)$ est orthogonale au plan $\mathscr{P}$.
    $\quad$
    $C$ est donc le projeté orthogonal du point $D$ sur le plan $\mathscr{P}$.
    $\quad$
    c. Une équation du plan $\mathscr{P}$ est donc de la forme $2x+y-z+d=0$.
    Le point $A$ appartient à ce plan. Ainsi $2+0-(-1)+d=0 \ssi d=-3$.
    Une équation cartésienne du plan $\mathscr{P}$ est $2x+y-z-3=0$.
    $\quad$
  2. a.
    $\begin{align*} CD&=\sqrt{2^2+1^2+(-1)^2} \\
    &=\sqrt{6}\end{align*}$
    $\quad$
    b. $C$ est le projeté orthogonal du point $D$ sur le plan $\mathscr{P}$ c’est donc l’unique point de ce plan situé à la distance $\sqrt{6}$ de $D$.
    Il n’existe donc pas de point $M$ du plan $\mathscr{P}$ différent de $C$ vérifiant $MD=\sqrt{6}$.
    $\quad$
  3. a. Soit $t\in \R$.
    $\begin{align*}2\times 0+(2+t)-(-1+t)+3&=2+t+1-t+3 \\
    &=0\end{align*}$
    Ainsi, le point $M(0:2+t;-1+t)$ appartient au plan $\mathscr{P}$ pour tout $t\in \R$.
    La droite $\Delta$ est incluse dans le plan $\mathscr{P}$.
    $\quad$
    b. On appelle $N$ le point de $\Delta$ associé à la valeur $-2$. Ainsi $N(0;0;-3)$.
    $\vect{ND}\begin{pmatrix}4\\-1\\1\end{pmatrix}$.
    Un vecteur directeur de $\Delta$ est $\vec{u}\begin{pmatrix} 0\\1\\1\end{pmatrix}$.
    Par conséquent $\vec{u}.\vect{ND}=0-1+1=0$.
    La droite $(ND)$ est donc perpendiculaire à la droite $\Delta$ en $N$.
    $N$ est le projeté orthogonal du point $D$ sur la droite $\Delta$.
    $H$ est donc bien le point de $\Delta$ associé à la valeur $t=-2$.
    $\quad$
    c. Ainsi :
    $\begin{align*} HD&=\sqrt{4^2+(-1)^2+1^2} \\
    &=\sqrt{18}\\
    &=3\sqrt{2}\end{align*}$
    $\quad$

 

Ex 3

Exercice 3

Partie A

  1. On obtient l’arbre pondéré suivant :
    $\quad$
    $\quad$
  2. a. $\left(A,\conj{A}\right)$ forme un système complet d’événements fini. D’après la formule des probabilités totales on a :
    $\begin{align*} p(T)&=p(A)p_A(T)+p\left(\conj{A}\right)p_{\conj{A}}(T)\\
    &=0,97x+0,043(1-x) \\
    &=0,043+0,927x\end{align*}$
    $\quad$
    b. On sait que $p(T)=0,2$.
    Par conséquent :
    $\begin{align*} 0,2=0,043+0,927x&\ssi 0,157=0,927x \\
    &\ssi x=\dfrac{157}{927}\end{align*}$.
    La probabilité que l’individu choisi soit allergique est donc environ égale à $0,169$.
    $\quad$
  3. On calcule :
    $\begin{align*} p_T(A)&=\dfrac{p(A\cap T)}{p(T)} \\
    &=\dfrac{p(A)p_A(T)}{p(T)} \\
    &\approx \dfrac{0,169\times 0,97}{0,2}\\
    &\approx 0,820\end{align*}$
    L’affirmation est donc vraie.
    $\quad$

Partie B

  1. On répète $150$ fois de façon indépendante la même expérience de Bernoulli de paramètre $0,08$.
    Ainsi $X$ suit la loi binomiale de paramètres $n=150$ et $p=0,08$.
  2. On veut calculer :
    $\begin{align*} p(X=20)&=\dbinom{150}{20}0,08^{20}\times 0,92^{130} \\
    &\approx 0,008\end{align*}$
    La probabilité que $20$ personnes exactement parmi les $150$ interrogées soient allergiques est environ égale à $0,008$.
    $\quad$
  3. D’après la calculatrice :
    $\begin{align*} p(X\pg 15)&=1-p(X\pp 14) \\
    &\approx 0,220\end{align*}$
    La probabilité qu’au moins $10 \%$ des personnes parmi les $150$ interrogées soient allergiques est environ égale à $0,220$.
    $\quad$

Ex 4

Exercice 4

Partie A

  1. Pour tout réel $x>0$ on a :
    $\begin{align*} g'(x)&=-\dfrac{2}{x^2}+\dfrac{2}{x^3}+\dfrac{1}{x} \\
    &=\dfrac{-2x+2+x^2}{x^3}\end{align*}$
    Or $x^3>0$ sur $]0;+\infty[$.
    Ainsi $g'(x)$ est du signe de $x^2-2x+2$.
    $\quad$
  2. Le discriminant de $x^2-2x+2$ est $\Delta=-4<0$.
    Le signe de ce trinôme est celui de son coefficient principal qui est $1>0$.
    Ainsi, pour tout réel $x>0$, $x^2-2x+2>0$.
    Donc $g'(x)>0$ et la fonction $g$ est strictement croissante sur $]0;+\infty[$.
    $\quad$
  3. La fonction $g$ est continue (car dérivable) et strictement croissante sur $]0;+\infty[$.
    $g(0,5)=\ln(0,5)= -\ln(2)<0$ et $g(1)=1>0$.
    D’après le théorème de la bijection (ou corollaire du théorème des valeurs intermédiaires) l’équation $g(x)=0$ admet une unique solution sur l’intervalle $[0,5;1]$.
    $\quad$
  4. La fonction $g$ est strictement croissante sur $]0;+\infty[$ et s’annule en $\alpha$.
    Ainsi, pour tout $x\in ]0;\alpha[$ on a $g(x)<g(\alpha)$ soit $g(x)<0$ et, pour tout $x>\alpha$ on a $g(x)>g(\alpha)$ doit $g(x)>0$.
    $\quad$

Partie B

  1. Pour tout réel $x>0$ on a :
    $\begin{align*} f\dsec(x)&=\e^x\left(\dfrac{1}{x}+\ln(x)\right)+\e^x\left(-\dfrac{1}{x^2}+\dfrac{1}{x}\right) \\
    &=\e^x\left(\dfrac{1}{x}+\ln(x)-\dfrac{1}{x^2}+\dfrac{1}{x}\right) \\
    &=\e^x\left(\dfrac{2}{x}-\dfrac{1}{x^2}+\ln(x)\right)\end{align*}$
    $\quad$
  2. a. On a ainsi, pour tout réel $x>0$, $f\dsec(x)=g(x)\e^x$.
    La fonction exponentielle est strictement positive sur $\R$. $f\dsec(x)$ est donc du signe de $g(x)$.
    On obtient alors le tableau de signes suivant :
    $\quad$

    $\quad$
    b. La fonction $f\dsec$ ne s’annule qu’une fois en changeant de signe en $\alpha$.
    $\mathscr{C}_f$ possède donc une unique point d’inflexion $A$ d’abscisse $\alpha$.
    $\quad$
    c. La fonction $f$ est donc concave sur $]0;+\alpha]$ et convexe sur $[\alpha;+\infty[$.
    $\quad$
  3. a. $\lim\limits_{x\to 0^+} \e^x=1$ et $\lim\limits_{x\to 0^+} \ln(x)=-\infty$
    Par conséquent $\lim\limits_{x\to 0+}f(x)=-\infty$
    $\lim\limits_{x\to +\infty} \e^x=+\infty$ et $\lim\limits_{x\to +\infty} \ln(x)=+\infty$
    Par conséquent $\lim\limits_{x\to v}f(x)=+\infty$
    $\quad$
    b.
    $g(\alpha)=0\ssi \ln(\alpha)=\dfrac{1}{\alpha^2}-\dfrac{2}{\alpha}$
    Ainsi :
    $\begin{align*} f'(\alpha)&=\e^{\alpha}\left(\dfrac{1}{\alpha}+\ln(\alpha)\right) \\
    &=\e^{\alpha}\left(\dfrac{1}{\alpha}+\dfrac{1}{\alpha^2}-\dfrac{2}{\alpha}\right) \\
    &=\e^{\alpha}\left(-\dfrac{1}{\alpha}+\dfrac{1}{\alpha^2}\right) \\
    &=\dfrac{\alpha}{\alpha^2}(-\alpha+1)\end{align*}$
    c. On a $0,5<\alpha<1$ donc $1-\alpha>0$.
    La fonction exponentielle est strictement positive sur $\R$ et $\alpha^2>0$.
    Ainsi $f'(\alpha)>0$.
    La fonction $f’$ admet un minimum en $\alpha$ et $f'(\alpha)>0$.
    Ainsi, pour tout réel $x>0$ on a $f'(x)>0$.
    $\quad$
    d. On en déduit donc le tableau de variations suivant :
    $\quad$
    $\quad$

 

Énoncé

Le candidat est invité à faire figurer sur la copie toute trace de recherche, même incomplète ou non fructueuse, qu’il aura développée.
La qualité de la rédaction, la clarté et la précision des raisonnements seront prises en compte dans l’appréciation de la copie. Les traces de recherche, même incomplètes ou infructueuses, seront valorisées.

Exercice 1     4 points

Cet exercice est un questionnaire à choix multiples.
Pour chacune des questions suivantes, une seule des quatre réponses proposées est exacte. Une réponse exacte rapporte un point. Une réponse fausse, une réponse multiple ou l’absence de réponse à une question ne rapporte ni n’enlève de point. Pour répondre, indiquer sur la copie le numéro de la question et la lettre de la réponse choisie. Aucune justification n’est demandée.

  1. On considère la fonction $f$ définie sur $\R$ par $f(x)=x\e^{x^2-3}$.
    Une des primitives $F$ de la fonction $f$ sur $\R$ est définie par :
    a. $F(x)=2x\e^{x^2-3}$ ;
    b. $F(x)=\left(2x^2+1\right)\e^{x^2-3}$ ;
    c. $F(x)=\dfrac{1}{2}x\e^{x^2-3}$ ;
    d. $F(x)=\dfrac{1}{2}\e^{x^2-3}$.
    $\quad$
  2. On considère la suite $\left(u_n\right)$ définie pour tout entier naturel $n$ par : $u_n=\e^{2n+1}$.
    La suite $\left(u_n\right)$ est :
    a. arithmétique de raison $2$ ;
    b. géométrique de raison $\e$ ;
    c. géométrique de raison $\e^2$ ;
    d. convergente vers $\e$.
    $\quad$

Pour les questions 3. et 4., on considère la suite $\left(u_n\right)$ définie sur $\N$ par :
$\hspace{1cm} u_0 = 15$ et pour tout entier naturel $n$ : $u_{n+1} = 1,2u_n + 12$.

  1. La fonction Python suivante, dont la ligne 4 est incomplète, doit renvoyer la plus petite valeur de l’entier $n$ telle que $u_n > 10~000$.
    $$\begin{array}{|l|}
    \hline
    \text{def seuil() :}\\
    \quad \text{n=0}\\
    \quad \text{u=15}\\
    \quad \text{while …}\\
    \qquad \text{n=n+1}\\
    \qquad \text{u=1.2*u+12}\\
    \qquad \text{return(n}\\
    \hline
    \end{array}$$
    À la ligne 4, on complète par :
    a. $\text{u <=10 000}$ ;
    b. $\text{u = 10 000}$ ;
    c. $\text{u > 10 000}$ ;
    d. $\text{n <= 10 000}$.
    $\quad$
  2. On considère la suite $\left(v_n\right)$ définie sur $\N$ par : $v_n=u_n+60$. La suite $\left(v_n\right)$ est :
    a. une suite décroissante ;
    b. une suite géométrique de raison $1,2$ ;
    c. une suite arithmétique de raison $60$ ;
    d. une suite ni géométrique ni arithmétique.
    $\quad$

$\quad$

Exercice 2     5 points

L’espace est rapporté à un repère orthonormé $\Oijk$.
On considère les points $A(1 ; 0 ;-1)$, $B(3 ;-1 ; 2)$, $C(2 ;-2 ;-1)$ et $D(4 ;-1 ;-2)$.
On note $\Delta$ la droite de représentation paramétrique $\begin{cases} x=0\\y=2+t\\z=-1+t\end{cases}$, avec $t\in \R$.

  1. a. Montrer que les points $A$, $B$ et $C$ définissent un plan que l’on notera $\mathcal{P}$.
    $\quad$
    b. Montrer que la droite $(CD)$ est orthogonale au plan $\mathcal{P}$. Sur le plan $\mathcal{P}$, que représente le point $C$ par rapport à $D$ ?
    $\quad$
    c. Montrer qu’une équation cartésienne du plan $\mathcal{P}$ est : $2x+y-z-3=0$.
    $\quad$
  2. a. Calculer la distance $CD$.
    $\quad$
    b. Existe-t-il un point $M$ du plan $\mathcal{P}$ différent de $C$ vérifiant $MD=\sqrt{6}$ ? Justifier la réponse.
    $\quad$
  3. a. Montrer que la droite $\Delta$ est incluse dans le plan $\mathcal{P}$.
    $\quad$
    Soit $H$ le projeté orthogonal du point $D$ sur la droite $\Delta$.
    b. Montrer que $H$ est le point de $\Delta$ associé à la valeur $t =-2$ dans la représentation paramétrique de $\Delta$ donnée ci-dessus.
    $\quad$
    c. En déduire la distance du point $D$ à la droite $\Delta$.
    $\quad$

$\quad$

Exercice 3     4 points

Les parties A et B sont indépendantes.
Les probabilités demandées seront données à $10^{-3}$ près.

Pour aider à la détection de certaines allergies, on peut procéder à un test sanguin dont le résultat est soit positif, soit négatif.

Dans une population, ce test donne les résultats suivants :

  • Si un individu est allergique, le test est positif dans $97 \%$ des cas ;
  • Si un individu n’est pas allergique, le test est négatif dans $95,7\%$ des cas.

Par ailleurs, $20 \%$ des individus de la population concernée présentent un test positif.

On choisit au hasard un individu dans la population, et on note :

  • $A$ l’événement « l’individu est allergique » ;
  • $T$ l’événement « l’individu présente un test positif ».

On notera $\conj{A}$ et $\conj{T}$ les événements contraires de $A$ et $T$.

On appelle par ailleurs $x$ la probabilité de l’événement $A$ : $x = p(A)$.

$\quad$

Partie A

  1. Reproduire et compléter l’arbre ci-dessous décrivant la situation, en indiquant sur chaque branche la probabilité correspondante.
    $\quad$

    $\quad$
  2. a. Démontrer l’égalité : $p(T)=0,927x+0,043$.
    $\quad$
    b. En déduire la probabilité que l’individu choisi soit allergique.
    $\quad$
  3. Justifier par un calcul l’affirmation suivante :
    « Si le test d’un individu choisi au hasard est positif, il y a plus de $80\%$ de chances que cet individu soit allergique ».
    $\quad$

Partie B :

On réalise une enquête sur les allergies dans une ville en interrogeant $150$ habitants choisis au hasard, et on admet que ce choix se ramène à des tirages successifs indépendants avec remise.
On sait que la probabilité qu’un habitant choisi au hasard dans cette ville soit allergique est égale à $0,08$.
On note $X$ la variable aléatoire qui à un échantillon de $150$ habitants choisis au hasard associe le nombre de personnes allergiques dans cet échantillon.

  1. Quelle est la loi de probabilité suivie par la variable aléatoire $X$ ? Préciser ses paramètres.
    $\quad$
  2. Déterminer la probabilité que 20 personnes exactement parmi les $150$ interrogées soient allergiques.
    $\quad$
  3. Déterminer la probabilité qu’au moins $10\%$ des personnes parmi les $150$ interrogées soient allergiques.
    $\quad$

$\quad$

Exercice 4     7 points

PARTIE A

On définit sur l’intervalle $]0;+\infty[$ la fonction $g$ par : $g(x)=\dfrac{2}{x}-\dfrac{1}{x^2}+\ln(x)$ où $\ln$ désigne la fonction logarithme népérien.
On admet que la fonction $g$ est dérivable sur $]0; +\infty[$ et on note $g’$ sa fonction dérivée.

  1. Montrer que pour $x>0$, le signe de $g'(x)$ est celui du trinôme du second degré $\left(x^2-2x+2\right)$.
    $\quad$
  2. En déduire que la fonction $g$ est strictement croissante sur $]0; +\infty[$.
    $\quad$
  3. Montrer que l’équation $g(x)=0$ admet une unique solution sur l’intervalle $[0,5 ; 1]$, que l’on notera $\alpha$.
    $\quad$
  4. On donne le tableau de signes de $g$ sur l’intervalle $]0; +\infty[$ :
    $\quad$

    $\quad$
    Justifier ce tableau de signes à l’aide des résultats obtenus aux questions précédentes.
    $\quad$

PARTIE B
On considère la fonction $f$ définie sur l’intervalle $]0; +\infty[$ par : $f(x)=\e^x\ln(x)$.
On note $C_f$ la courbe représentative de $f$ dans un repère orthonormé.

  1. On admet que la fonction $f$ est deux fois dérivable sur $]0; +\infty[$ , on note $f’$ sa fonction dérivée, $f\dsec$ sa fonction dérivée seconde et on admet que :
    pour tout nombre réel $x > 0,~f'(x)=\e^x\left(\dfrac{1}{x}+\ln(x)\right)$
    Démontrer que, pour tout nombre réel $x > 0$, on a : $f\dsec(x)=\e^x\left(\dfrac{2}{x}-\dfrac{1}{x^2}+\ln(x)\right)$.
    $\quad$
  2. On pourra remarquer que pour tout réel $x>0$, $f\dsec(x) = \e^x\times g(x)$, où $g$ désigne la fonction étudiée dans la partie A.
    a. Dresser le tableau de signes de la fonction $f\dsec(x)$ sur $]0; +\infty[$. Justifier.
    $\quad$
    b. Justifier que la courbe $C_f$ admet un unique point d’inflexion $A$.
    $\quad$
    c. Étudier la convexité de la fonction $f$ sur l’intervalle $]0; +\infty[$. Justifier.
    $\quad$
  3. a. Calculer les limites de $f$ aux bornes de son ensemble de définition.
    $\quad$
    b. Montrer que $f'(x)(\alpha) =\dfrac{\e^{\alpha}}{\alpha^2}(1-\alpha)$. On rappelle que $\alpha$ est l’unique solution de l’équation $g(\alpha) = 0$.
    $\quad$
    c. Démontrer que $f'(\alpha)> 0$ et en déduire le signe de $f'(x)$ pour $x$ appartenant à $]0; +\infty[$.
    $\quad$
    d. En déduire le tableau de variations complet de la fonction $f$ sur $]0; +\infty[$.
    $\quad$

$\quad$

 

Bac – Spécialité mathématiques – Nouvelle Calédonie – sujet 2 – 29 août 2023

Nouvelle Calédonie – 29 août 2023

Spécialité maths – Sujet 2 – Correction

L’énoncé de ce sujet de bac est disponible ici.

Ex 1

Exercice 1

  1. Les points $F$ et $K$ appartiennent au plan $(EHG)$, ne sont pas confondus et le point $C$ n’appartient pas à ce plan.
    Ainsi $C$, $F$ et $K$ définissent bien un plan.
    $\quad$
  2. a. $K$ est le milieu de $[HG]$ et $HG=1$ donc $KG=0,5$.
    $[GF]$ et $[GC]$ sont des arêtes du cube. Donc $GF=GC=1$.
    $\quad$
    b. Le triangle $FGC$ est rectangle en $G$.
    L’aire du triangle $FGC$ est donc :
    $\begin{align*} A_{FGC}&=\dfrac{GF\times GC}{2} \\
    &=\dfrac{1}{2} \text{u.a.}\end{align*}$
    $\quad$
    c. Le volume du tétraèdre $FGCK$ est
    $\begin{align*} V_{FGCK}&=\dfrac{A_{FGC}\times KG}{3} \\
    &=\dfrac{\dfrac{1}{2}\times \dfrac{1}{2}}{3} \\
    &=\dfrac{1}{12} \text{u.v.}\end{align*}$
    $\quad$
  3. a. On a $C(1;1;0)$, $F(0;1;1)$ et $K(1;0,5;1)$.
    Donc $\vect{CF}\begin{pmatrix}-1\\0\\1\end{pmatrix}$ et $\vect{CK}\begin{pmatrix}0\\-0,5\\1\end{pmatrix}$
    Ces deux vecteurs ne sont pas colinéaires car ils n’ont pas la même composante nulle (ou car, d’après la question 1, ils définissent un plan).
    $\vec{n}.\vect{CF}=-1+0+1=0$ et $\vec{n}.\vect{CK}=0-1+1=0$.
    $\vec{n}$ est orthogonal à deux vecteurs non colinéaires du plan $(CFK)$. Il est donc normal à ce plan.
    $\quad$
    b. Une équation cartésienne du plan $(CFK)$ est donc de la forme $x+2y+z+d=0$.
    $C(1;1;0)$ appartient à ce plan. Par conséquent $1+2+0+d=0\ssi d=-3$.
    Une équation cartésienne du plan $(CFK)$ est par conséquent $x+2y+z-3=0$.
    $\quad$
  4. La droite $\Delta$ passe par $G(1;1;1)$ et admet comme vecteur directeur le vecteur $\vec{n}$.
    Une représentation paramétrique de la droite $\Delta$ est donc $\begin{cases} x=1+t\\y=1+2t\\z=1+t\end{cases} \quad (t\in \R)$.
    $\quad$
  5. a. On note $L(x;y;z)$.
    Les coordonnées de $L$ sont solution du système
    $\begin{align*} \begin{cases} x+2y+z-3=0\\x=1+t\\y=1+2t\\z=1+t\end{cases}&\ssi \begin{cases} 1+t+2+4t+1+t-3=0\\x=1+t\\y=1+2t\\z=1+t\end{cases} \\
    &\ssi \begin{cases} 6t+1=0\\x=1+t\\y=1+2t\\z=1+t\end{cases}\\
    &\ssi \begin{cases} t=-\dfrac{1}{6}\\[3mm]x=\dfrac{5}{6}\\[3mm]y=\dfrac{2}{3}\\[3mm]\dfrac{5}{6}\end{cases}\end{align*}$
    Ainsi les coordonnées du point $L$ sont $\left(\dfrac{5}{6};\dfrac{2}{3};\dfrac{5}{6}\right)$.
    $\quad$
    b. On a alors $\vect{LG}\begin{pmatrix}\dfrac{1}{6}\\[3mm]\dfrac{1}{3}\\[3mm]\dfrac{1}{6}\end{pmatrix}$
    Donc :
    $\begin{align*} LG&=\sqrt{\dfrac{1}{6^2}+\dfrac{1}{3^2}+\dfrac{1}{6^2}} \\
    &=\sqrt{\dfrac{1}{6}} \\
    &=\dfrac{\sqrt{6}}{6}\end{align*}$
    $\quad$
  6. On a
    $\begin{align*}V_{FGCK}=\dfrac{1}{12}&\ssi \dfrac{A_{CFK}\times LG}{3}=\dfrac{1}{12} \\
    &\ssi A_{CFK}\times \dfrac{\sqrt{6}}{6}=\dfrac{1}{4} \\
    &\ssi A_{CFK}=\dfrac{\sqrt{6}}{4} \text{u.a.}\end{align*}$.
    L’aire du triangle $CFK$ est donc égale à $\dfrac{\sqrt{6}}{4} $ u.a.

Ex 2

Exercice 2

  1. Pour tout réel $x\pg 0$ on a
    $\begin{align*} f(x)&=x\e^{-x} \\
    &=x\times \dfrac{1}{\e^x} \\
    &=\dfrac{x}{\e^x}\end{align*}$
    Par croissances comparées, $\lim\limits_{x\to +\infty} \dfrac{\e^x}{x}=+\infty$ donc $\lim\limits_{x\to +\infty} f(x)=0$.
    La droite d’équation $y=0$ est par conséquent une asymptote à la courbe $\mathcal{C}_f$ en $+\infty$.
    $\quad$
  2. D’après l’énoncé $f$ est dérivable sur $\R_+$.
    Pour tout réel $x\pg 0$ on a
    $\begin{align*} f'(x)&=\e^{-x}-x\e^{-x} \\
    &=(1-x)\e^{-x}\end{align*}$
    $\quad$
  3. La fonction exponentielle est strictement positive sur $\R$.
    Le signe de $f'(x)$ ne dépend donc que de celui de $1-x$.
    Or $1-x=0 \ssi x=1$ et $1-x>0\ssi x<1$.
    On obtient alors le tableau de variations suivant :
    $\quad$
    $\quad$
  4. La fonction $f$ est continue (car dérivable) et strictement croissante sur $[0;1]$.
    $f(0)=0$ et $f(1)=\e^{-1}\approx 0,3679$. Donc $\dfrac{367}{1~000}\in \left]0;\e^{-1}\right[$.
    D’après le théorème de la bijection (ou corollaire du théorème des valeurs intermédiaires), l’équation $f(x)=\dfrac{367}{1~000}$ admet une unique solution sur $]0;1[$.
    $\quad$
    La fonction $f$ est continue (car dérivable) et strictement décroissante sur $[1;+\infty[$.
    $\lim\limits_{x\to +\infty} f(x)=0$ et $f(1)=\e^{-1}\approx 0,3679$. Donc $\dfrac{367}{1~000}\in \left]0;\e^{-1}\right[$.
    D’après le théorème de la bijection (ou corollaire du théorème des valeurs intermédiaires), l’équation $f(x)=\dfrac{367}{1~000}$ admet une unique solution sur $]1;+\infty[$.
    $\quad$
    Finalement, l’équation $f(x)=\dfrac{367}{1~000}$ admet exactement deux solutions sur $[0;+\infty[$.
    $\quad$
  5. La fonction exponentielle est strictement positive sur $\R$.
    Le signe de $f\dsec(x)$ ne dépend donc que de celui de $x-2$.
    $x-2=0\ssi x=2$ et $x-2>0\ssi x>2$.
    La fonction $f$ est donc concave sur $[0;2]$ et convexe sur $[2;+\infty[$.
    $\quad$
  6. a. Une équation de la droite $T_a$ est :
    $\begin{align*}y=f'(a)(x-a)+f(a)&\ssi y=(1-a)\e^{-a}(x-a)+a\e^{-a} \\
    &\ssi y=(1-a)\e^{-a}x-a\e^{-a}+a^2\e^{-a}+a\e^{-a} \\
    &\ssi y=(1-a)\e^{-a}x+a^2\e^{-a}\end{align*}$
    $\quad$
    b. L’ordonnée à l’origine de $T_a$  est $a^2\e^{-a}$.
    Donc $g(a)=a^2\e^{-a}$.
    $\quad$
    c. On considère la fonction $g$ définie sur $[0;+\infty[$ par $g(x)=x^2\e^{-x}$.
    La fonction $g$ est dérivable sur $[0;+\infty[$ en tant que produit de fonctions dérivables sur cet intervalle.
    Pour tout réel $x\pg 0$ on a
    $\begin{align*} g'(x)&=2x\e^{-x}-x^2\e^{-x} \\
    &=x(2-x)\e^{-x} \\
    &=-xf\dsec(x)\end{align*}$
    Ainsi, sur $[0;+\infty[$ $g'(x)$ et $f\dsec(x)$ sont de signes contraires.
    D’après la question 5., $g(a)$ est maximale quand $x=2$ c’est-à-dire quand $A$ est un point d’inflexion de $\mathcal{C}_f$.
    $\quad$

 

Ex 3

Exercice 3

  1. On a
    $\begin{align*} u_1&=\dfrac{-u_0-4}{u_0+3} \\
    &=-\dfrac{4}{3}\end{align*}$
    $\begin{align*} u_2&=\dfrac{-u_1-4}{u_1+3} \\
    &=-\dfrac{8}{5}\end{align*}$
    $\quad$
  2. On peut écrire $$\begin{array}{|l|}
    \hline
    \text{def terme(n):}
    \quad \text{u = 0} \\
    \quad \text{for i in range(n):}\\
    \qquad \text{u = (-u – 4)/(u + 3)}\\
    \quad \text{return(u)}\\
    \hline
    \end{array}$$
    $\quad$
  3. La fonction $f$ est dérivable sur $]-3;+\infty[$ en tant que quotient de fonctions dérivables dont le dénominateur ne s’annule pas sur cet intervalle.
    Pour tout réel $x>-3$ on a :
    $\begin{align*} f'(x)&=\dfrac{-(x+3)-(-x-4)}{(x+3)^2} \\
    &=\dfrac{-x-3+x+4}{(x+3)^2} \\
    &=\dfrac{1}{(x+3)^2}\\
    &>0\end{align*}$
    La fonction $f$ est donc strictement croissante sur $]-3;+\infty[$.
    $\quad$
  4. Pour tout $n\in \N$ on pose $P(n):~-2<u_{n+1} \pp u_n$.
    Initialisation : $u_0=0$ et $u_1=-\dfrac{4}{3}$ donc $-2<u_1\pp u_0$ et $P(0)$ est vraie.
    $\quad$
    Hérédité : Soit $n\in \N$. On suppose $P(n)$ vraie.
    On a $-2<u_{n+1}\pp u_n$.
    La fonction $f$ est strictement croissante sur $]-3;+\infty[$.
    Par conséquent $f(-2)<f\left(u_{n+1}\right)\pp f\left(u_n\right)$
    Donc $-2<u_{n+2}\pp u_{n+1}$ et $P(n+1)$ est vraie.
    $\quad$
    Conclusion : La propriété est vraie au rang $0$ et est héréditaire.
    Par conséquent, pour tout $n\in \N$, $-2<u_{n+1}\pp u_n$.
    $\quad$
  5. La suite $\left(u_n\right)$ est décroissante et minorée par $-2$.
    Elle converge donc.
    $\quad$
  6. a. On a $v_0=\dfrac{1}{2}$.
    $\quad$
    b. Soit $n\in \N$.
    $\begin{align*} v_{n+1}-v_n&=\dfrac{1}{u_{n+1}+2}-\dfrac{1}{u_n+2} \\
    &=\dfrac{1}{\dfrac{-u_n-4}{u_n+3}+2}-\dfrac{1}{u_n+2} \\
    &=\dfrac{1}{\dfrac{-u_n-4+2u_n+6}{u_n+3}}-\dfrac{1}{u_n+2} \\
    &=\dfrac{1}{\dfrac{u_n+2}{u_n+3}}-\dfrac{1}{u_n+2} \\
    &=\dfrac{u_n+3}{u_n+2}-\dfrac{1}{u_n+2} \\
    &=\dfrac{u_n+2}{u_n+2}\\
    &=1\end{align*}$
    La suite $\left(v_n\right)$ est donc arithmétique de raison $1$.
    $\quad$
    c. Ainsi, pour tout $n\in \N$, on a $v_n=\dfrac{1}{2}+n$.
    Or $v_n=\dfrac{1}{u_n+2}\ssi u_n+2=\dfrac{1}{v_n} \ssi u_n=\dfrac{1}{0,5+n}-2$.
    $\quad$
    d. $\lim\limits_{n\to +\infty} \dfrac{1}{n+0,5}=0$ donc $\lim\limits_{n\to +\infty} u_n=-2$.
    $\quad$

Ex 4

Exercice 4

  1. On a $P_A(F)=\dfrac{25}{75}$.
    Réponse b
    $\quad$
  2. On a
    $\begin{align*} P(A\cup F)&=\dfrac{75+80}{200} \\
    &=\dfrac{155}{200}  \\
    &=\dfrac{31}{40}\end{align*}$
    Réponse c
    $\quad$
  3. On appelle $B$ l’événement “le bus est en panne” et $T$ l’événement ‘le train est en panne”.
    On veut calculer :
    $\begin{align*}p_1&=P(B\cup T)\\
    &=P(B)+P(T)-P(B\cap T)\\
    &=b+t-P(B)P(T) \qquad \text{(indépendance)}\\
    &=b+t-bt\end{align*}$
    Réponse d
    $\quad$
  4. Albert peut se rendre à son travail si le train et le bus ne sont pas en panne. Donc
    $\begin{align*} p_2&=P\left(\conj{B\cap T}\right) \\
    &=1-P(B\cap T) \\
    &=1-P(B)P(T) \qquad \text{(indépendance)}\\
    &=1-bt\end{align*}$
    Réponse b
    $\quad$
  5. On appelle $X$ la variable aléatoire égale au nombre de FACE.
    On effectue $n$ expériences identiques de Bernoulli de paramètre $x$.
    $X$ suit donc la loi binomiale de paramètre $n$ et $x$.
    Ainsi :
    $\begin{align*} P(X\pg 1)&=1-P(X=0) \\
    &=1-(1-x)^n\end{align*}$
    Réponse d
    $\quad$

Énoncé

La qualité de rédaction, la clarté et la précision des raisonnements seront prises en compte dans l’appréciation de la copie. Les traces de recherche, même incomplètes ou infructueuses, seront valorisées.

Exercice 1     5 points

On considère le cube $ABCDEFGH$ d’arête $1$ représenté ci-dessous.

On note $K$ le milieu du segment $[HG]$.
On se place dans le repère orthonormé $\left(A;\vect{AD},\vect{AB},\vect{AE}\right)$.

  1. Justifier que les points $C$, $F$ et $K$ définissent un plan.
    $\quad$
  2. a. Donner, sans justifier, les longueurs $KG$, $GF$ et $GC$.
    $\quad$
    b. Calculer l’aire du triangle $FGC$.
    $\quad$
    c. Calculer le volume du tétraèdre $FGCK$.
    On rappelle que le volume $V$ d’un tétraèdre est donné par :
    $$V=\dfrac{1}{3}\mathcal{B}\times h$$
    où $\mathcal{B}$ est l’aire d’une base et $h$ la hauteur correspondante.
    $\quad$
  3. a. On note $\vec{n}$ le vecteur de coordonnées $\begin{pmatrix}1\\2\\1\end{pmatrix}$.
    Démontrer que $\vec{n}$ est normal au plan $(CFK)$.
    $\quad$
    b. En déduire qu’une équation cartésienne du plan $(CFK)$ est :
    $$x +2y + z-3 = 0$$
    $\quad$
  4. On note $\Delta$ la droite passant par le point $G$ et orthogonale au plan $(CFK)$.
    Démontrer qu’une représentation paramétrique de la droite $\Delta$ est :
    $$\begin{cases}x=1+t\\y=1+2t\\z=1+t\end{cases}\quad (t\in \R)$$
    $\quad$
  5. Soit $L$ le point d’intersection entre la droite $\Delta$ et le plan $(CFK)$.
    a. Déterminer les coordonnées du point $L$.
    $\quad$
    b. En déduire que $LG = \dfrac{\sqrt{6}}{6}$.
    $\quad$
  6. En utilisant la question 2., déterminer la valeur exacte de l’aire du triangle $CFK$.
    $\quad$

$\quad$

Exercice 2     5 points

On considère la fonction $f$ , définie sur $[0 ;+\infty[$ par : $$f(x) = x\e^{-x}$$
On note $\mathcal{C}_f$ sa courbe représentative dans un repère orthonormé du plan.
On admet que $f$ est deux fois dérivable sur $[0 ;+\infty[$.
On note $f’$ sa dérivée et $f\dsec$ sa dérivée seconde.

  1. En remarquant que pour tout x dans $[0 ;+\infty[$, on a $f(x) =\dfrac{x}{\e^x}$ , démontrer que la courbe $\mathcal{C}_f$ possède une asymptote en $+\infty$ dont on donnera une équation.
    $\quad$
  2. Démontrer que pour tout réel $x$ appartenant à $[0 ;+\infty[$ : $$f'(x) = (1-x)\e^{-x}$$
    $\quad$
  3. Dresser le tableau de variations de $f$ sur $[0 ;+\infty[$, sur lequel on fera figurer les valeurs aux bornes ainsi que la valeur exacte de l’extremum.
    $\quad$
  4. Déterminer, sur l’intervalle $[0 ;+\infty[$, le nombre de solutions de l’équation : $$f(x) = \dfrac{367}{1~000}$$
    $\quad$
  5. On admet que pour tout $x$ appartenant à $[0 ;+\infty[$ : $$f\dsec(x) = \e^{-x}(x-2)$$
    Étudier la convexité de la fonction $f$ sur l’intervalle $[0 ;+\infty[$.
    $\quad$
  6. Soit $a$ un réel appartenant à $[0 ;+\infty[$ et $A$ le point de la courbe $\mathcal{C}_f$ d’abscisse $a$.
    On note $T_a$ la tangente à $\mathcal{C}_f$ en $A$.
    On note $H_a$ le point d’intersection de la droite $T_a$ et de l’axe des ordonnées.
    On note $g(a)$ l’ordonnée de $H_a$.
    La situation est représentée sur la figure ci-dessous.
    $\quad$

    $\quad$
    a. Démontrer qu’une équation réduite de la tangente $T_a$ est :
    $$y=\left((1-a)\e^{-a}\right).x+a^2\e^{-a}$$
    $\quad$
    b. En déduire l’expression de $g(a)$.
    $\quad$
    c. Démontrer que $g(a)$ est maximum lorsque $A$ est un point d’inflexion de la courbe $C_f$.
    Les traces de recherche, même incomplètes ou infructueuses, seront valorisées.
    $\quad$

$\quad$

Exercice 3     5 points

On considère la suite $\left(u_n\right)$ telle que $u_0 = 0$ et pour tout entier naturel $n$ : $$u_{n+1} =\dfrac{-u_n-4}{u_n +3}$$
On admet que $u_n$ est défini pour tout entier naturel $n$.

  1. Calculer les valeurs exactes de $u_1$ et $u_2$.
    $\quad$
  2. On considère la fonction terme ci-dessous écrite de manière  incomplète en langage Python :
    $$\begin{array}{|l|}
    \hline
    \text{def terme(n):}\\
    \quad \text{u = …}\\
    \quad \text{for i in range(n):}\\
    \qquad \text{u = …}\\
    \quad \text{return(u)}\\
    \hline
    \end{array}$$
    On rappelle qu’en langage Python, « $\text{i in range(n)}$ » signifie que $\text{i}$ varie de $\text{0}$ à $\text{n-1}$.
    Recopier et compléter le cadre ci-dessus de sorte que, pour tout entier naturel $n$, l’instruction $\text{terme(n)}$ renvoie la valeur de $u_n$.
    $\quad$
  3. Soit la fonction $f$ définie sur $]-3 ;+\infty[$ par : $$f(x) = \dfrac{-x-4}{x+3}$$
    Ainsi, pour tout entier naturel $n$, on a $u_{n+1} = f\left(u_n\right)$.
    Démontrer que la fonction $f$ est strictement croissante sur $]-3 ;+\infty[$.
    $\quad$
  4. Démontrer par récurrence que pour tout entier naturel $n$ :
    $$−2 < u_{n+1} \pp u_n$$
    $\quad$
  5. En déduire que la suite $\left(u_n\right)$ est convergente.
    $\quad$
  6. Soit la suite $\left(v_n\right)$ définie pour tout entier naturel $n$ par : $$vn = \dfrac{1}{u_n+2}$$
    a. Donner $v_0$.
    $\quad$
    b. Démontrer que la suite $\left(v_n\right)$ est arithmétique de raison $1$.
    $\quad$
    c. En déduire que pour tout entier naturel $n$ : $$u_n =\dfrac{1}{n+0,5}-2$$
    $\quad$
    d. Déterminer la limite de la suite $\left(u_n\right)$.
    $\quad$

$\quad$

Exercice 4     5 points

Cet exercice est un questionnaire à choix multiples.
Pour chacune des questions suivantes, une seule des quatre réponses proposées est exacte.
Pour répondre, indiquer sur la copie le numéro de la question et la lettre de la réponse choisie.
Aucune justification n’est demandée.
Une réponse fausse, une absence de réponse, ou une réponse multiple, ne rapporte ni n’enlève de point.

L’énoncé ci-dessous est commun aux questions 1. et 2.

Les $200$ adhérents d’un club sont des filles ou des garçons. Ces adhérents pratiquent l’aviron ou le basket selon la répartition figurant dans le tableau ci-dessous.
$$\begin{array}{|c|c|c|c|}
\hline
&\text{Aviron}&\text{Basket}&\text{Total}\\
\hline
\text{Filles}& 25& 80& 105\\
\hline
\text{Garçon}& 50&45&95\\
\hline
\text{Total}& 75& 125& 200\\
\hline
\end{array}$$
On choisit un adhérent au hasard et on considère les évènements suivants :
$F$ : l’adhérent est une fille.  $\qquad A$ : l’adhérent pratique l’aviron.

  1. La probabilité de $F$ sachant $A$ est égale à :
    a. $\dfrac{25}{100_{\phantom{1}}}$
    b. $\dfrac{25}{75_{\phantom{1}}}$
    c. $\dfrac{25}{105_{\phantom{1}}}$
    d. $\dfrac{75}{105_{\phantom{1}}}$
    $\quad$
  2. La probabilité de l’événement $A\cup F$ est égale à :
    a. $\dfrac{9}{10_{\phantom{1}}}$
    b. $\dfrac{1}{8_{\phantom{1}}}$
    c. $\dfrac{31}{40_{\phantom{1}}}$
    d. $\dfrac{5}{36_{\phantom{1}}}$
    $\quad$
    $$\begin{array}{c} \ast\\[-1cm]\ast\ast\end{array}$$

L’énoncé ci-dessous est commun aux questions 3. et 4.

Pour se rendre à son travail, Albert peut utiliser au choix le bus ou le train.

La probabilité que le bus soit en panne est égale à $b$.
La probabilité que le train soit en panne est égale à $t$.
Les pannes de bus et de train surviennent de façon indépendante.

  1. La probabilité $p_1$, que le bus ou le train soient en panne est égale à :
    a. $p_1 = bt$
    b. $p_1 = 1-bt$
    c. $p_1 = b+t$
    d. $p_1 = b + t-bt$
    $\quad$
  2. La probabilité p2 que Albert puisse se rendre à son travail est égale à :
    a. $p_1 = bt$
    b. $p_1 = 1-bt$
    c. $p_1 = b+t$
    d. $p_1 = b + t-bt$
    $\quad$
    $$\begin{array}{c} \ast\\[-1cm]\ast\ast\end{array}$$

 

  1. On considère une pièce de monnaie pour laquelle la probabilité d’obtenir FACE est égale à $x$. On lance la pièce $n$ fois. Les lancers sont indépendants.
    La probabilité $p$ d’obtenir au moins une fois FACE sur les $n$ lancers est égale à :
    a. $p = x^n$
    b. $p = (1- x)^n$
    c. $p = 1-x^n$
    d. $p = 1-(1-x)^n$
    $\quad$

$\quad$

 

Bac – Spécialité mathématiques – La Réunion – sujet 2 – 29 mars 2023

La Réunion – 29 mars 2023

Spécialité maths – Sujet 2 – Correction

L’énoncé de ce sujet de bac est disponible ici.

Ex 1

Exercice 1

Partie A

  1. On obtient l’arbre pondéré suivant :
    $\quad$

    $\quad$
  2. $\left(R,\conj{R}\right)$ forme un système complet d’événements fini. D’après la formule des probabilités totales :
    $\begin{align*} P(S)=P(S\cap R)+P\left(S\cap \conj{R}\right)&\ssi 0,82=P(R)P_R(S)+P\left(\conj{R}\right)P_{\conj{R}}(S) \\
    &\ssi 0,82=0,2\times 0,9+0,8x \\
    &\ssi 0,64=0,8x \\
    &\ssi x=0,8\end{align*}$
    $\quad$
  3. On veut calculer :
    $\begin{align*} P_S(R)&=\dfrac{P(S\cap R)}{P(S)} \\
    &=\dfrac{P(R)P_R(S)}{P(S)} \\
    &=\dfrac{0,2\times 0,9}{0,82} \\
    &=\dfrac{9}{41} \\
    &\approx 0,22\end{align*}$
    La probabilité que le client ait acheté un matelas RESSORTS sachant qu’il a été satisfait de son achat est environ égal à $0,22$.
    $\quad$

Partie B

  1. a. $X$ suit la loi binomiale de paramètres $n=5$ et $p=0,82$.
    $\quad$
    b. La probabilité qu’au plus trois clients soient satisfaits de leur achat est $$P(X\pp 3)\approx 0,222$$
    $\quad$
  2. a. On répète $n$ fois de façon indépendante la même expérience de Bernoulli de paramètre $p=0,82$.
    On appelle $Y$ la variable aléatoire qui donne le nombre de clients satisfaits de leur achat parmi ces $n$ clients.
    $Y$ suit donc la loi binomiale de paramètres $n$ et $p=0,82$.
    Ainsi,
    $\begin{align*} p_n&=P(Y=n) \\
    &=0,82^n\end{align*}$
    $\quad$
    b.
    $\begin{align*} p_n<0,01 &\ssi 0,82^n <0,01 \\
    &\ssi n\ln(0,82) < \ln(0,01) \\
    &\ssi n>\dfrac{\ln(0,01)}{\ln(0,82)}\qquad \text{(car $\ln(0,82)<0$)}\end{align*} $
    Or $\dfrac{\ln(0,01)}{\ln(0,82)}\approx 23,2$.
    Ainsi $p_n<0,01$ si, et seulement si, $n\pg 24$.
    La probabilité que tous les clients soient satisfaits de leur achat est inférieure à $1\%$ dès qu’il y a au moins $24$ clients.
    $\quad$

Ex 2

Exercice 2

  1. On a
    $\begin{align*} u_1&=\dfrac{6u_0+2}{u_0+5} \\
    &=\dfrac{48+2}{13 }\\
    &=\dfrac{50}{13}\end{align*}$
    $\quad$
  2. a. La fonction $f$ est dérivable sur $[0;+\infty[$ en tant que quotient de fonctions dérivables dont le dénominateur ne s’annule pas sur cet intervalle.
    Pour tout $x>0$ on a
    $\begin{align*} f'(x)&=\dfrac{6(x+5)-(6x+2)}{(x+5)^2} \\
    &=\dfrac{28}{(x+5)^2}\\
    &>0\end{align*}$
    La fonction $f$ est donc strictement croissante sur l’intervalle $[0;+\infty[$.
    $\quad$
    $f(2)=\dfrac{14}{7}=2$.
    La fonction $f$ étant strictement croissante sur $[0;+\infty[$, pour tout $x>2$ on a $f(x)>f(2)$ soit $f(x)>2$.
    $\quad$
    b. Pour tout $n\in \N$ on a $P(n):~u_n>2$.
    Initialisation : $u_0=8>2$. Donc $P(0)$ est vraie.
    $\quad$
    Hérédité : Soit $n\in \N$. On suppose que $P(n)$ est vraie.
    Donc $u_n>2$. D’après la question 2.a, $f\left(u_n\right) > 2$ soit $u_{n+1}>2$.
    Par conséquent $P(n+1)$ est vraie.
    $\quad$
    Conclusion : La propriété est vraie au rang $0$ et est héréditaire.
    Pour tout $n\in \N$, $u_n>2$.
    $\quad$
  3. a. Pour tout $n\in \N$ on a $u_{n+1}-u_n=\dfrac{\left(2-u_n\right)\left(u_n+1\right)}{u_n+5}$.
    D’après la question précédente, pour tout $n\in \N$, $u_n>2$.
    Ainsi $2-u_n<0$, $u_n+1>0$ et $u_n+5>0$.
    Donc $u_{n+1}-u_n<0$ et la suite $\left(u_n\right)$ est décroissante.
    $\quad$
    b. La suite $\left(u_n\right)$ est décroissante et minorée par $2$; elle converge donc .
  4. a. $v_0=\dfrac{8-2}{8+1}=\dfrac{2}{3}$
    $\quad$
    b. Soit $n\in \N$.
    $\begin{align*} v_{n+1}&=\dfrac{u_{n+1}-2}{u_{n+1}+1} \\
    &=\dfrac{\dfrac{6u_n+2}{u_n+5}-2}{\dfrac{6u_n+2}{u_n+5}+1} \\
    &=\dfrac{~\dfrac{6u_n+2-2u_n-10}{u_n+5}~}{\dfrac{6u_n+2+u_n+5}{u_n+5}} \\
    &=\dfrac{4u_n-8}{7u_n+7} \\
    &=\dfrac{4}{7}\times \dfrac{u_n-2}{u_n+1}\\
    &=\dfrac{4}{7}v_n\end{align*}$
    La suite $\left(v_n\right)$ est donc géométrique de raison $\dfrac{4}{7}$ et de premier terme $v_0=\dfrac{2}{3}$.
    $\quad$
    c. Pour tout $n\in \N$, on a $v_n=\dfrac{2}{3}\left(\dfrac{4}{7}\right)^n$.
    $-1<\dfrac{4}{7}<1$ donc $\lim\limits_{n\to +\infty} v_n=0$.
    $\quad$
    Pour tout $n\in \N$ on a
    $\begin{align*} v_n=\dfrac{u_n-2}{u_n+1}&\ssi v_n\left(u_n+1\right)=u_n-2 \\
    &\ssi u_nv_n+v_n=u_n-2\\
    &\ssi u_nv_n-u_n=-2-v_n\\
    &\ssi u_n\left(v_n-1\right)=-2-v_n \\
    &\ssi u_n=\dfrac{-2-v_n}{v_n-1}\end{align*}$
    Par conséquent $\lim\limits_{n\to +\infty} u_n=\lim\limits_{n\to +\infty}\dfrac{-2-v_n}{v_n-1}=2$.
    $\quad$
  5. On a $u_{13}\approx 2,0014>2,001$ et $u_{14}\approx 2,000~8<2,001$.
    La commande $\texttt{seuil(2.001)}$ renverra donc la valeur $14$.
    Il s’agit du rang à partir duquel tous les termes de la suite prendront des valeurs inférieures ou égales à $2,001$.

Ex 3

Exercice 3

  1. Une représentation paramétrique de la droite $(d)$ est $$\begin{cases} x=1\\y=1+2t\\z=-t\end{cases} \qquad \forall t\in \R$$
    $\quad$
  2. Un vecteur normal au plan $\mathscr{P}$ est $\vec{w}\begin{pmatrix}1\\4\\2\end{pmatrix}$.
    $\vec{u}$ et $\vec{w}$ ne sont pas colinéaires car ils n’ont pas la même composante nulle.
    Ainsi $(d)$ et $\mathscr{P}$ sont sécants.
    $1-4+2+1=4-4=0$ : le point de coordonnées $(1;-1;1)$ appartient au plan $\mathscr{P}$.
    En prenant $t=-1$ dans la représentation paramétrique de $(d)$ on obtient le point de coordonnées $(1;-1;1)$.
    Ainsi la droite $(d)$ et le plan $\mathscr{P}$ sont sécants en un point $B$ de coordonnées $(1;-1;1)$.
    $\quad$
  3. a. $\vect{AC}\begin{pmatrix} 0\\-2\\-1\end{pmatrix}$ et $\vect{AB}\begin{pmatrix} 0\\-2\\1\end{pmatrix}$.
    $\dfrac{-2}{-2}=1$ et $\dfrac{-1}{1}=-1$ donc $\vect{AB}$ et $\vect{AC}$ ne sont pas colinéaires.
    Par conséquent $A$, $B$ et $C$ définissent bien un plan.
    $\quad$
    b. $\vec{n}.\vect{AC}=0+0+0=0$ et $\vec{n}.\vect{AB}=0+0+0$.
    Le vecteur $\vec{n}$ est orthogonal à deux vecteurs non colinéaires du plan $(ABC)$.
    Donc $\vec{n}$ est un vecteur normal au plan $(ABC)$.
    $\quad$
    c. Une équation cartésienne du plan $(ABC)$ est donc de la forme $x+d=0$.
    $A(1;1;0)$ appartient à ce plan. Par conséquent $1+d=0 \ssi d=-1$.
    Une équation cartésienne du plan $(ABC)$ est $x-1=0$.
    $\quad$
  4. a.
    $\begin{align*} AB&=\sqrt{0^2+(-2)^2+1^2}\\
    &=\sqrt{5}\end{align*}$
    $\begin{align*} AC&=\sqrt{0^2+(-2)^2+(-1)^2}\\
    &=\sqrt{5}\end{align*}$
    Ainsi $AB=AC$ et le triangle $ABC$ est isocèle en $A$.
    $\quad$
    b. $H$ est le milieu de $[BC]$. Il a donc pour coordonnées $\left(\dfrac{1+1}{2};\dfrac{-1-1}{2};\dfrac{1-1}{2}\right)$ soit $(1;-1;0)$.
    Donc $\vect{AH}\begin{pmatrix} 0\\-2\\0\end{pmatrix}$
    Donc :
    $\begin{align*} AH&=\sqrt{0^2+(-2)^1+0} \\
    &=2\end{align*}$
    $\vect{BC}\begin{pmatrix}0\\0\\-2\end{pmatrix}$
    On a donc également $BC=2$.
    Le triangle $ABC$ est isocèle en $A$ donc $[AH]$ est à la fois une médiane, une médiatrice, une hauteur et une bissectrice du triangle.
    L’aire du triangle $ABC$ est :
    $\begin{align*} \mathscr{A}&=\dfrac{AH\times BC}{2} \\
    &=2\text{ u.a.}\end{align*}$
    $\quad$
  5. a. $\vect{BD}\begin{pmatrix} -1\\0\\0\end{pmatrix}$
    Ainsi $\vec{n}=-\vect{BD}$.
    $\vect{BD}$ est donc normal au plan $(ABC)$.
    Par conséquent $(BD)$ est une hauteur de la pyramide $ABCD$.
    $\quad$
    b. $\quad$
    $\begin{align*} BD&=\sqrt{1^2+0^2+0^2}\\
    &=1\end{align*}$
    Par conséquent :
    $\begin{align*} V&=\dfrac{1}{3}\times \mathscr{A}\times BD\\
    &=\dfrac{2}{3} \text{ u.v.}\end{align*}$
    $\quad$

 

Ex 4

Exercice 4

  1. La fonction $f$ est dérivable sur $\R$ en tant que produit de fonctions dérivables sur $\R$.
    Pour tout réel $x$ on a :
    $\begin{align*} f'(x)&=2\e^x+2x\e^x \\
    &=2(x+1)\e^x\end{align*}$
    La fonction exponentielle est strictement positive sur $\R$.
    Le signe de $f'(x)$ ne dépend donc que de celui de $x+1$.
    Or $x+1=0\ssi x=-1$ et $x+1>0\ssi x>-1$.
    La fonction $f$ est donc strictement décroissante sur $]-\infty;-1]$ et strictement croissante sur $[-1;+\infty[$.
    De plus $f(-1)=-2\e^{-1} \approx -0,736$.
    $\quad$
    La fonction $f$ est continue (car dérivable) et strictement décroissante sur $]-\infty;-1]$
    Par croissances comparées $\lim\limits_{x\to -\infty} f(x)=0>-\dfrac{73}{100}$ et $f(-1)<-\dfrac{73}{100}$
    D’après le théorème de la bijection, l’équation $f(x)=-\dfrac{73}{100}$ possède une unique solution sur $]-\infty;-1]$.
    $\quad$
    La fonction $f$ est continue (car dérivable) et strictement croissante sur $[-1;+\infty[$
    $f(-1)<-\dfrac{73}{100}$ et $\lim\limits_{x\to +\infty} f(x)=+\infty$ (produit de deux fonctions tendant vers $+\infty$).
    D’après le théorème de la bijection, l’équation $f(x)=-\dfrac{73}{100}$ possède une unique solution sur $[-1;+\infty[$.
    $\quad$
    L’équation $f(x)=-\dfrac{73}{100}$ possède donc exactement deux solutions sur $\R$.
    Réponse c
    $\quad$
  2. $\lim\limits_{x\to -\infty} x+1=-\infty$ et $\lim\limits_{x\to -\infty} \e^x=0^+$.
    Par conséquent $\lim\limits_{x\to -\infty} g(x)=-\infty$.
    Réponse a
    $\quad$
  3. La fonction $h$ est dérivable sur $\R$ en tant que produit de fonctions dérivables sur $\R$.
    Pour tout réel $x$ on a :
    $\begin{align*} h'(x)&=4\e^{2x}+2(4x-16)\e^{2x} \\
    &=(4+8x-32)\e^{2x} \\
    &=(8x-28)\e^{2x} \\
    &=4(2x-7)\e^{2x}\end{align*}$
    La fonction $h’$ est dérivable sur $\R$ en tant que produit de fonctions dérivables sur $\R$.
    Pour tout réel $x$ on a :
    $\begin{align*} h\dsec(x)&=4\left(2\e^{2x}+2(2x-7)\e^{2x}\right) \\
    &=8(1+2x-7)\e^{2x} \\
    &=8(2x-6)\e^{2x}\end{align*}$
    $h\dsec(x)>0 \ssi 2x-6>0 \ssi x>3$ et $\dsec(x)=0 \ssi 2x-6=0\ssi x=3$.
    La fonction $h\dsec$ s’annule en changeant de signe en $3$.
    Le point d’abscisse $3$ est donc un point d’inflexion pour la courbe $\mathscr{C}_h$.
    Réponse b
    $\quad$
  4. La fonction $k$ est dérivable sur $]0;+\infty[$ en tant que somme de fonctions dérivables sur cet intervalle.
    Pour tout réel $x>0$ on a $k'(x)=\dfrac{3}{x}-1$
    Une équation de $T$ est $y=k'(\e)(x-\e)+k(\e)$.
    Par conséquent $k'(\e)=\dfrac{3-\e}{\e}$ et $k(\e)=3-\e$.
    Une équation de $T$ est donc $y=\dfrac{3-\e}{\e}(x-\e)+3-\e$
    Soit $y=\dfrac{3-\e}{\e}x$
    Réponse b
    $\quad$
  5. $\left(\ln(x)\right)^2+10\ln(x)+21=0 \ssi \begin{cases} X^2+10X+21=0 \\X=\ln(x)\end{cases}$
    Le discriminant de l’équation $X^2+10X+21=0$ est $\Delta=16$.
    Elle possède donc deux solutions $\dfrac{-10-\sqrt{16}}{2}=-7$ et $\dfrac{-10+\sqrt{16}}{2}=-3$.
    $\ln(x)=-7 \ssi x=\e^{-7}$
    $\ln(x)=-3\ssi x=\e^{-3}$.
    Par conséquent $\e^{-7}$ et $\e^{-3}$ sont les solutions de l’équation $\left(\ln(x)\right)^2+10\ln(x)+21=0$.
    Réponse c
    $\quad$

 

Énoncé

La qualité de rédaction, la clarté et la précision des raisonnements seront prises en compte dans l’appréciation de la copie. Les traces de recherche, même incomplètes ou infructueuses, seront valorisées.

Exercice 1     5 points

Un commerçant vend deux types de matelas: matelas RESSORTS et matelas MOUSSE.
On suppose que chaque client achète un seul matelas.

On dispose des informations suivantes :

  • $20\%$ des clients achètent un matelas RESSORTS. Parmi eux, $90\%$ sont satisfaits de leur achat.
  • $82\%$ des clients sont satisfaits de leur achat.

Les deux parties peuvent être traitées de manière indépendante.

Partie A

On choisit au hasard un client et on note les évènements :

  • $R$ : : « le client achète un matelas RESSORTS »,
  • $S$ : « le client est satisfait de son achat ».

On note $x = P_{\conj{R}}(S)$, où $P_{\conj{R}}(S)$ désigne la probabilité de $S$ sachant que $R$ n’est pas réalisé.

  1. Recopier et compléter l’arbre pondéré ci-dessous décrivant la situation.
    $\quad$
    $\quad$
  2. Démontrer que $x = 0,8$.
    $\quad$
  3. On choisit un client satisfait de son achat.
    Quelle est la probabilité qu’il ait acheté un matelas RESSORTS ?
    On arrondira le résultat à $10^{-2}$.

Partie B

  1. On choisit $5$ clients au hasard. On considère la variable aléatoire $X$ qui donne le nombre de clients satisfaits de leur achat parmi ces $5$ clients.
    a. On admet que $X$ suit une loi binomiale. Donner ses paramètres.
    $\quad$
    b. Déterminer la probabilité qu’au plus trois clients soient satisfaits de leur achat.
    On arrondira le résultat à $10^{-3}$.
  2. Soit $n$ un entier naturel non nul.
    On choisit à présent $n$ clients au hasard. Ce choix peut être assimilé à un tirage au sort avec remise.
    a. On note $p_n$ la probabilité que les $n$ clients soient tous satisfaits de leur achat.
    Démontrer que $p_n = 0,82^n$.
    $\quad$
    b. Déterminer les entiers naturels $n$ tels que $p_n < 0,01$.
    Interpréter dans le contexte de l’exercice.
    $\quad$

$\quad$

Exercice 2     5 points

On considère la suite $\left(u_n\right)$ définie par $u_0 = 8$ et, pour tout entier naturel $n$, $$u_{n +1} = \dfrac{6u_n+2}{u_n +5}$$

  1. Calculer $u_1$.
    $\quad$
  2. Soit $f$ la fonction définie sur l’intervalle $[0;+\infty[$ par : $$f(x) = \dfrac{6x+2 }{x+5}$$
    Ainsi, pour tout entier naturel $n$, on a : $u_{n+1}=f\left(u_n\right)$.
    a. Démontrer que la fonction $f$ est strictement croissante sur l’intervalle $[0;+\infty[$.
    En déduire que pour tout réel $x > 2$, on a $f(x) > 2$.
    $\quad$
    b. Démontrer par récurrence que, pour tout entier naturel $n$, on a $u_n > 2$.
    $\quad$
  3. On admet que, pour tout entier naturel $n$, on a : $$u_{n+1}-u_n = \dfrac{\left(2-u_n\right)\left(u_n+1\right)}{u_n +5}$$
    a. Démontrer que la suite $\left(u_n\right)$ est décroissante.
    $\quad$
    b. En déduire que la suite $\left(u_n\right)$ est convergente.
    $\quad$
  4. On définit la suite $\left(v_n\right)$ pour tout entier naturel par: $$v_n = \dfrac{u_n-2}{u_n+1}$$
    a. Calculer $v_0$.
    $\quad$
    b. Démontrer que $\left(v_n\right)$ est une suite géométrique de raison $\dfrac{4}{7}$.
    $\quad$
    c. Déterminer, en justifiant, la limite de $\left(v_n\right)$.
    En déduire la limite de $\left(u_n\right)$.
    $\quad$
  5. On considère la fonction Python $\text{seuil}$ ci-dessous, où $\text{A}$ est un nombre réel strictement plus grand que $2$.
    $$\begin{array}{|l|}
    \hline
    \text{def seuil (A) :}\\
    \quad \text{n = 0}\\
    \quad \text{u = 8}\\
    \quad \text{while u > A :}\\
    \qquad \text{u = (6*u + 2) / (u + 5)}\\
    \qquad \text{n = n + 1}\\
    \quad \text{return n}\\
    \hline
    \end{array}$$
    Donner, sans justification, la valeur renvoyée par la commande $\text{seuil (2.001)}$ puis interpréter cette valeur dans le contexte de l’exercice.
    $\quad$

$\quad$

Exercice 3     5 points

On se place dans l’espace rapporté à un repère orthonormé $\Oijk$.
On considère le point $A(1;1;0)$ et le vecteur $\vec{u}\begin{pmatrix}0\\2\\- 1\end{pmatrix}$.
On considère le plan $\mathcal{P}$ d’équation : $x+4y+2z+1 = 0$.

  1. On note $(d)$ la droite passant par A et dirigée par le vecteur $\vec{u}$.
    Déterminer une représentation paramétrique de $(d)$.
    $\quad$
  2. Justifier que la droite $(d)$ et le plan $\mathcal{P}$ sont sécants en un point $B$ dont les coordonnées sont $(1;-1;1)$.
    $\quad$
  3. On considère le point $C(1;-1;-1)$.
    a. Vérifier que les points $A$, $B$ et $C$ définissent bien un plan.
    $\quad$
    b. Montrer que le vecteur  $\vec{n}\begin{pmatrix}1\\0\\0\end{pmatrix}$ est un vecteur normal au plan $(ABC)$.
    $\quad$
    c. Déterminer une équation cartésienne du plan $(ABC)$.
    $\quad$
  4. a. Justifier que le triangle $ABC$ est isocèle en $A$.
    $\quad$
    b. Soit $H$ le milieu du segment $[BC]$.
    Calculer la longueur $AH$ puis l’aire du triangle $ABC$.
    $\quad$
  5. Soit $D$ le point de coordonnées $(0;-1;1)$.
    a. Montrer que la droite $(BD)$ est une hauteur de la pyramide $ABCD$.
    $\quad$
    b. Déduire des questions précédentes le volume de la pyramide $ABCD$.
    $\quad$
    On rappelle que le volume $V$ d’une pyramide est donné par: $$V = \dfrac13 \mathcal{B} \times h$$
    où $\mathcal{B}$ est l’aire d’une base et $h$ la hauteur correspondante.
    $\quad$

$\quad$

Exercice 4     5 points

Cet exercice est un questionnaire à choix multiples. Pour chacune des questions suivantes, une seule des quatre réponses proposées est exacte. Pour répondre, indiquer sur la copie le numéro de la question et la lettre de la réponse choisie. Aucune justification n’est demandée. Une réponse fausse, une absence de réponse, ou une réponse multiple, ne rapporte ni n’enlève de point.

  1. On considère la fonction $f$ définie sur $\R$ par $f(x) = 2x\e^x$.
    Le nombre de solutions sur $\R$ de l’équation $f(x) = -\dfrac{73}{100}$ est égal à :
    a. $0$
    b. $1$
    c. $2$
    d. une infinité.
    $\quad$
  2. On considère la fonction $g$ définie sur $\R$ par : $$g(x) = \dfrac{x+ 1}{\e^x}$$
    La limite de la fonction $g$ en $- \infty$ est égale à :
    a. $-\infty$
    b. $+\infty$
    c. $0$
    d. elle n’existe pas.
    $\quad$
  3. On considère la fonction $h$ définie sur $\R$ par: $$h(x) = (4x-16)\e^{2x}$$
    On note $\mathcal{C}_h$ la courbe représentative de $h$ dans un repère orthogonal.
    On peut affirmer que:
    a. $h$ est convexe sur $\R$.
    b. $\mathcal{C}_h$ possède un point d’inflexion en $x = 3$.
    c. $h$ est concave sur $\R$.
    d. $\mathcal{C}_h$ possède un point d’inflexion en $x = 3,5$.
    $\quad$
  4. On considère la fonction $k$ définie sur l’intervalle $]0; +\infty[$ par : $$k(x) = 3 \ln (x)-x$$
    On note $\mathcal{C}$ la courbe représentative de la fonction $k$ dans un repère orthonormé.
    On note $T$ la tangente à la courbe $\mathcal{C}$ au point d’abscisse $x = \e$.
    Une équation de $T$ est:
    a. $y = (3-\e)x$
    b. $y = \left(\dfrac{3-\e}{\e}\right)x$
    c. $y = \left(\dfrac{3}{\e}- 1\right)x + 1$
    d. $y = (\e-1)x + 1$
    $\quad$
  5. On considère l’équation $\left(\ln (x)\right)^2+10\ln(x)+21 = 0$, avec $x \in ]0;+\infty[$.
    Le nombre de solutions de cette équation est égal à :
    a. $0$
    b. $1$
    c. $2$
    d. une infinité.
    $\quad$

$\quad$

Bac – Spécialité mathématiques – La Réunion – sujet 1 – 28 mars 2023

La Réunion – 28 mars 2023

Spécialité maths – Sujet 1 – Correction

L’énoncé de ce sujet de bac est disponible ici.

Ex 1

Exercice 1

Partie A

  1. On obtient l’arbre pondéré suivant :
    $\quad$

    $\quad$
  2. On a :
    $\begin{align*} P(A)&=P\left(\left(D_1\cap A\right)\cup\left(\conj{D_1}\cap D_2\cap A\right)\right) \\
    &=P\left(D_1\cap A\right)+P\left(\conj{D_1}\cap D_2\cap A\right) \qquad \text{(incompatibilité)}\\
    &=P\left(D_1\right) P_{D_1}(A)+P\left(\conj{D_1}\right)P_{\conj{D_1}}\left(D_2\right)P_{\conj{D_1}\cap D_2}(A) \\
    &=0,4\times 0,3+0,6\times 0,7\times 0,2 \\
    &=0,204\end{align*}$
    $\quad$
  3. On veut calculer :
    $\begin{align*}P_A\left(D_1\right) &=\dfrac{P\left(A\cap D_1\right)}{P(A)} \\
    &=\dfrac{P\left(D_1\right)P_{D_1}(A)}{P(A)} \\
    &=\dfrac{0,4\times 0,3}{0,204} \\
    &=\dfrac{10}{17} \\
    &\approx 0,588\end{align*}$
    La probabilité que la personne ait décroché au premier appel sachant qu’elle a acheté le produit est environ égale à$0,588$.
    $\quad$

Partie B

  1. a. $X$ suit la loi binomiale de paramètres $n=30$ et $p=0,204$.
    $\quad$
    b. On veut calculer :
    $\begin{align*} P(X=6)&=\dbinom{30}{6}0,204^6\times (1-0,204)^{24} \\
    &\approx 0,179\end{align*}$
    La probabilité qu’exactement $6$ personnes de l’échantillon achètent le produit est environ égale à $0,179$.
    $\quad$
    c. L’espérance de $X$ est :
    $\begin{align*} E(X)&=30\times 0,204 \\
    &=6,12\end{align*}$
    Cela signifie donc, qu’en moyenne, sur un échantillon de $30$ personnes  $6,12$ achètent le produit.
    $\quad$
  2. On effectue $n$ fois de façon indépendante la même expérience de Bernoulli de paramètre $p=0,204$.
    On appelle $Y$ la variable aléatoire qui donne le nombre de personnes de l’échantillon qui achètent le produit.
    $Y$ suit donc la loi binomiale de paramètres $n$ et $p=0,204$.
    On veut déterminer le plus petit entier naturel $n$ tel que :
    $\begin{align*} P(Y\pg 1)\pg 0,99 &\ssi 1-P(Y=0)\pg 0,99 \\
    &\ssi P(Y=0) \pp 0,01 \\
    &\ssi (1-0,204)^n \pp 0,01\\
    &\ssi 0,796^n\pp 0,01 \\
    &\ssi n\ln(0,796) \pp \ln(0,01) \\
    &\ssi n\pg \dfrac{\ln(0,01)}{\ln(0,796)} \qquad \text{(car $\ln(0,796)<0$)} \end{align*}$
    Or $\dfrac{\ln(0,01)}{\ln(0,796)} \approx 20,2$
    Il faut donc l’échantillon contienne au moins $21$ personnes.
    $\quad$

Ex 2

Exercice 2

  1. $\lim\limits_{x\to 0} 3x+1=1$
    Par croissances comparées, $\lim\limits_{x\to 0} x\ln(x)=0$
    Par conséquent, $\lim\limits_{x\to 0} f(x)=1$.
    $\quad$
    Pour tout $x>0$ on a $f(x)=x\left(3+\dfrac{1}{x}-2\ln(x)\right)$.
    $\lim\limits_{x\to +\infty} \dfrac{1}{x}=0$ et $\lim\limits_{x\to +\infty} \ln(x)=+\infty$ par conséquent $\lim\limits_{x\to +\infty} 3+\dfrac{1}{x}-2\ln(x)=-\infty$.
    Ainsi $\lim\limits_{x\to +\infty} f(x)=-\infty$.
    $\quad$
  2. a. $f$ est derivable sur $\R_+^*$ par hypothèse. Pour tout réel $x>0$ on a :
    $\begin{align*} f'(x)&=3-2\ln(x)-2x\times \dfrac{1}{x} \\
    &=3-2\ln(x)-2 \\
    &=1-2\ln(x)\end{align*}$
    $\quad$
    b. $1-2\ln(x)=0 \ssi \ln(x)=\dfrac{1}{2} \ssi x=\e^{1/2}$
    $1-2\ln(x)>0 \ssi \ln(x)<\dfrac{1}{2} \ssi x<\e^{1/2}$
    On obtient ainsi le tableau de variations suivant :
    $\quad$

    $\begin{align*} m&=f\left(\e^{1/2}\right) \\
    &=3\e^{1/2}+1-2\e^{1/2}\times \dfrac{1}{2} \\
    &=2\e^{1/2}+1\end{align*}$
    $\quad$
  3. a. La fonction $f$ est strictement croissante sur l’intervalle $\left]0;\e^{1/2}\right]$ et $\lim\limits_{x\to 0} f(x)=1$.
    Ainsi, pour tout $x\in \left]0;\e^{1/2}\right]$ on a $f(x)>1$.
    L’équation $f(x)=0$ n’admet donc aucune solution sur cet intervalle.
    $\quad$
    La fonction $f$ est continue et strictement décroissante sur l’intervalle $\left[\e^{1/2};+\infty\right[$.
    $f\left(\e^{1/2}\right)=2\e^{1/2}+1>0$ et $\lim\limits_{x\to +\infty} f(x)=-\infty$.
    D’après le théorème de la bijection (ou corollaire du théorème des valeurs intermédiaires) l’équation $f(x)=0$ admet une unique solution $\alpha$ sur $\left[\e^{1/2};+\infty\right[$.
    $\quad$
    Finalement, l’équation $f(x)=0$ admet une unique solution sur $]0;+\infty[$.
    $\quad$
    b. D’après le tableau de variations de la fonction $f$ et la question précédente :
    $\bullet~f(x)>0$ si $x\in ]0;\alpha[$ ;
    $\bullet~f(\alpha)=0$ ;
    $\bullet~f(x)<0$ si $x\in ]\alpha;+\infty[$.
    $\quad$
  4. $F$ est une primitive de $f$ sur $]0;+\infty[$.
    $f$ est donc la dérivée de $F$ sur cet intervalle.
    Or $f(x)>0$ sur $\left]\e^{1/2};\alpha\right[$.
    La fonction $F$ est donc strictement croissante sur cet intervalle.
    L’affirmation est fausse.
    $\quad$
  5. a. Pour tout réel $x>0$ on a $f\dsec(x)=-\dfrac{2}{x}<0$.
    La fonction $f$ est donc concave sur $]0;+\infty[$.
    La courbe $\mathscr{C}_f$ est donc située sous ses tangentes.
    $\quad$
    b. Une équation de $\mathscr{T}$ est $y=f'(1)(x-1)+f(1)$.
    Or $f'(1)=1$ et $f(1)=4$.
    Une équation de $\mathscr{T}$ est donc $y=x-1+4$ soit $y=x+3$.
    $\quad$
    c. D’après la question 5.a. on a donc en particulier :
    $\begin{align*} f(x)\pp x+3 &\ssi 3x+1-2x\ln(x) \pp x+3 \\
    &\ssi -2x\ln(x) \pp -2x+2 \\
    &\ssi \ln(x)\pg 1-\dfrac{1}{x}\end{align*}$

Continue reading

Bac – Spécialité mathématiques – Amérique du Nord – sujet 2 – 28 mars 2023

Amérique du Nord – 28 mars 2023

Spécialité maths – Sujet 2 – Correction

L’énoncé de ce sujet de bac est disponible ici.

Ex 1

Exercice 1

Partie A

  1. La fonction $f’$ semble être positive sur $]-\infty;0,3]$ et sur $[2,5;+\infty[$ et négative sur $[0,3;2,5]$.
    Par conséquent $f$ semble croissante sur $]-\infty;0,3]$ et sur $[2,5;+\infty[$ et décroissante sur $[0,3;2,5]$.
    $\quad$
  2. La fonction $f’$ semble strictement croissante sur $]-\infty;-1]$ et $[2;+\infty[$ et strictement décroissante sur $[-1;2]$.
    La fonction $f$ semble être convexe sur $]-\infty;-1]$ et sur $[2;+\infty[$.
    $\quad$

Partie B

  1. a. D’après la limite des termes de plus haut degré, $\lim\limits_{x\to +\infty} x^2-5x+6=\lim\limits_{x\to +\infty} x^2=+\infty$.
    De plus $\lim\limits_{x\to +\infty} \e^x=+\infty$.
    Par conséquent $\lim\limits_{x\to +\infty} f(x)=+\infty$.
    $\quad$
    b. Pour tout réel $x$ on a $f(x)=x^2\e^x-5x\e^x+6\e^x$.
    $\lim\limits_{x\to -\infty} \e^x=0$ et, par croissances comparées, $\lim\limits_{x\to -\infty} x\e^x=0$ et $\lim\limits_{x\to -\infty} x^2\e^x=0$.
    Ainsi $\lim\limits_{x\to -\infty} f(x)=0$.
    $\quad$
  2. La fonction $f$ est dérivable sur $\R$ en tant que somme et produit de fonctions dérivables sur $\R$.
    Pour tout réel $x$ on a :
    $\begin{align*} f'(x)&=(2x-5)\e^x+\left(x^2-5x+6\right)\e^x \\
    &=\left(2x-5+x^2-5x+6\right)\e^x \\
    &=\left(x^2-3x+1\right)\e^x\end{align*}$.
    $\quad$
  3. La fonction exponentielle est strictement positive sur $\R$.
    Par conséquent $f'(x)$ est du même signe que $x^2-3x+1$.
    Il s’agit d’un polynôme du second degré dont le discriminant est $\Delta=5>0$.
    Ses racines sont donc $\dfrac{3-\sqrt{5}}{2}$ et $\dfrac{3+\sqrt{5}}{2}$.
    $\quad$
    De plus son coefficient principal est $1>0$.
    Par conséquent :
    $\bullet~f'(x)<0$ sur $\left]\dfrac{3-\sqrt{5}}{2};\dfrac{3+\sqrt{5}}{2}\right[$ ;
    $\bullet~f'(x)=0$ si $x\in \acco{\dfrac{3-\sqrt{5}}{2};\dfrac{3+\sqrt{5}}{2}}$ ;
    $\bullet~f'(x)>0$ sur $\left]-\infty;\dfrac{3-\sqrt{5}}{2}\right[$ et $\left]\dfrac{3+\sqrt{5}}{2};+\infty\right[$.
    La fonction $f$ est donc strictement croissante sur $\left]-\infty;\dfrac{3-\sqrt{5}}{2}\right[$ et $\left]\dfrac{3+\sqrt{5}}{2};+\infty\right[$ et strictement décroissante sur $\left]\dfrac{3-\sqrt{5}}{2};\dfrac{3+\sqrt{5}}{2}\right[$ .
    $\quad$
  4. Une équation de $(\mathscr{T})$ est $y=f'(0)x+f(0)$.
    Or $f(0)=6$ et $f'(0)=1$.
    Une équation de $(\mathscr{T})$ est donc $y=x+6$.
    $\quad$
  5. a. La fonction exponentielle est strictement positive sur $\R$.
    Par conséquent $f\dsec(x)$ est du signe de $(x+1)(x-2)$.
    $x+1=0\ssi x=-1$ et $x+1>0\ssi x>-1$
    $x-2=0\ssi x=2$ et $x-2>0 \ssi x>2$.
    Par conséquent $f\dsec(x)<0 \ssi x\in ]-1;2[$.
    La fonction $f$ est concave sur $[-1;2]$ et convexe sur $]-\infty;-1]$ et sur $[2;+\infty[$.
    $\quad$
    b. La fonction $f$ est concave sur $[-1;2]$. Sa courbe représentative est donc située sous ses tangentes sur cet intervalle.
    Or $0$ appartient à $[-1;2]$.
    Par conséquent $f(x)\pp x+6$.
    $\quad$

 

Ex 2

Exercice 2

  1. Pour tout entier naturel $n$ on a donc $a_{n+1}=(1-0,15)a_n+0,1b_n$ soit $a_{n+1}=0,85a_n+0,1b_n$ et $b_{n+1}=0,15a_n+(1-0,1)b_n$ soit $b_{n+1}=0,15a_n+0,9b_n$.
    Par conséquent
    $\begin{align*} a_1&=0,85\times 1~700+0,1\times 1~300\\
    &=1~575\end{align*}$
    $\begin{align*} b_1&=0,15\times 1~700+0,9\times 1~300\\
    &=1~425\end{align*}$
    En 2024, le club A comptera $1~575$ membres et le club B $1~425$.
    $\quad$
  2. Durant l’étude aucun sportif ne quitte le groupe.
    Par conséquent, pour tout $n\in \N$, on a $a_n+b_n=3~000$.
    $\quad$
  3. Pour tout $n\in \N$ on a $a_{n+1}=0,85a_n+0,1b_n$ et $a_n+b_n=3~000$.
    Par conséquent :
    $\begin{align*} a_{n+1}&=0,85a_n+0,1\left(3~000-a_n\right) \\
    &=0,85a_n+300-0,1a_n \\
    &=0,75a_n+300\end{align*}$
    $\quad$
  4. a. Pour tout $n\in \N$ on pose $P(n):~1~200\pp a_{n+1}\pp a_n\pp 1~700$.
    Initialisation : $a_0=1~700$ et $a_1=1~575$. Donc $P(0)$ est vraie.
    $\quad$
    Hérédité : Soit $n\in \N$. On suppose que $P(n)$ est vraie.
    $1~200\pp a_{n+1}\pp a_n\pp 1~700$
    donc
    $900\pp 0,75a_{n+1}\pp 0,75a_n\pp 1~275$
    Par conséquent $1~200 \pp 0,75a_{n+1}+300\pp 0,75a_n+300\pp 1~575$.
    Donc $1~200\pp a_{n+2} \pp a_{n+1} \pp 1~575\pp 1~700$.
    Ainsi $P(n+1)$ est vraie.
    $\quad$
    Conclusion : La propriété est vraie au rang $0$ et est héréditaire.
    Pour tout $n\in \N$, $1~200\pp a_{n+1}\pp a_n\pp 1~700$.
    $\quad$
    b. La suite $\left(a_n\right)$ est décroissante et minorée par $1~200$ ; elle converge donc.
    $\quad$
  5. a. Soit $n\in \N$.
    $\begin{align*} v_{n+1}&=a_{n+1}-1~200 \\
    &=0,75a_n+300-1~200\\
    &=0,75a_n-900 \\
    &=0,75\left(a_n-1~200\right) \\
    &=0,75v_n\end{align*}$
    La suite $\left(v_n\right)$ est donc géométrique de raison $0,75$ et de premier terme $v_0=a_0-1~200$ soit $v_0=500$.
    $\quad$
    b. Pour tout $n\in \N$ on a donc $v_n=500\times 0,75^n$.
    $\quad$
    c. Pour tout $n\in \N$ on a :
    $\begin{align*} a_n&=v_n+1~200 \\
    &=500\times 0,75^n+1~200\end{align*}$
    $\quad$
  6. a. $-1<0,75<1$ donc $\lim\limits_{n\to +\infty} 0,75^n=0$. Ainsi $\lim\limits_{n\to +\infty} a_n=1~200$.
    $\quad$
    b. Sur le long terme, le club A comptera ainsi $1~200$ membres.
    $\quad$
  7. a. On peut écrire
    $$\begin{array}{|l|}
    \hline
    \texttt{def seuil() :}\\
    \hspace{0.8cm} \texttt{n = 0}\\
    \hspace{0.8cm} \texttt{A = 1700}\\
    \hspace{0.8cm} \texttt{while A >= 1280 :}\\
    \hspace{1.6cm} \texttt{n = n + 1}\\
    \hspace{1.6cm} \texttt{A = 0.75 * A + 300}\\
    \hspace{0.8cm} \texttt{return n}\\
    \hline
    \end{array}$$
    $\quad$
    b. On veut déterminer le plus petit entier naturel $n$ tel que :
    $\begin{align*} a_n< 1~280 &\ssi 500\times 0,75^n+1200< 1~280 \\
    &\ssi 500\times 0,75^n< 80 \\
    &\ssi 0,75^n < 0,16\\
    &\ssi n\ln(0,75)<\ln(0,16) \\
    &\ssi n>\dfrac{\ln(0,16)}{\ln(0,75)} \qquad \text{(car $\ln(0,75)<0$)}\end{align*}$
    Or $\dfrac{\ln(0,16)}{\ln(0,75)} \approx 6,4$.
    Ainsi l’appel de la fonction $\texttt{seuil}$ renverra $7$.
    $\quad$

Ex 3

Exercice 3

  1. a. $\vect{EF}\begin{pmatrix}-4\\4\\2\end{pmatrix}$ et $\vect{FG}\begin{pmatrix} 4\\0\\-4\end{pmatrix}$
    $\quad$
    b. Ces deux vecteurs ne sont pas colinéaires puisqu’ils n’ont pas la même composante nulle.
    Ainsi les points $E$, $F$ et $G$ ne sont pas alignés.
    $\quad$
  2. a. Une représentation paramétrique de la droite $(FG)$ est donc $$\begin{cases} x=-1+4t\\y=2\\z=1-4t\end{cases} \qquad \forall t\in \R$$
    $\quad$
    b. $-1+4t=2\ssi 4t=3\ssi t=\dfrac{3}{4}$
    $4t-1=-2 \ssi -1+4t=2\ssi t=\dfrac{3}{4}$
    Donc en prenant $t=\dfrac{3}{4}$ dans la représentation paramétrique de la droite $(FG)$ on retrouve les coordonnées de point $H$.
    De plus $\vect{EH}\begin{pmatrix}-1\\4\\-1\end{pmatrix}$.
    Ainsi $\vect{EH}.\vect{FG}=-4+0+4=0$.
    Les droites $(EH)$ et $(FG)$ sont perpendiculaires en $H$.
    $H$ est le projeté orthogonal du point $E$ sur la droite $(FG)$.
    $\quad$
    c. On a :
    $\begin{align*} FG&=\sqrt{4^2+0+(-4)^2} \\
    &=\sqrt{32} \\
    &=4\sqrt{2}\end{align*}$
    $\begin{align*} EH&=\sqrt{(-1)^2+4^2+(-1)^2} \\
    &=\sqrt{18} \\
    &=3\sqrt{2}\end{align*}$
    L’aire du triangle $EFG$ est donc égale à :
    $\begin{align*} \mathscr{A}&=\dfrac{EH\times FG}{2} \\
    &=\dfrac{4\sqrt{2}\times 3\sqrt{2}}{2} \\
    &=12 \text{ cm}^2\end{align*}$
    $\quad$
  3. a. $\vec{n}.\vect{EF}=-8+4+4=0$
    $\vec{n}.\vect{FG}=8+0-8=0$
    Le vecteur $\vec{n}$ est orthogonal à deux vecteurs non colinéaires du plan $(EFG)$.
    Il est donc normal au plan $(EFG)$.
    $\quad$
    b. Une équation cartésienne du plan $(EFG)$ est de la forme $2x+y+2z+d=0$.
    $E(3;-2;-1)$ appartient à ce plan.
    Ainsi $6-2-2+d=0 \ssi d=-2$.
    Une équation cartésienne du plan $(EFG)$ est donc $2x+y+2z-2=0$.
    $\quad$
    c. Une représentation paramétrique de la droite $(d)$ est donc $$\begin{cases}x=3+2k\\y=1+k\\z=5+2k\end{cases} \qquad \forall k\in \R$$
    $\quad$
    d. On résout le système :
    $\begin{align*} \begin{cases}x=3+2k\\y=1+k\\z=5+2k\\2x+y+2z-2=0\end{cases}&\ssi \begin{cases}x=3+2k\\y=1+k\\z=5+2k\\6+4k+1+k+10+4k-2=0\end{cases} \\
    &\ssi \begin{cases}x=3+2k\\y=1+k\\z=5+2k\\9k=-15\end{cases} \\
    &\ssi \begin{cases}k=-\dfrac{5}{3}\\[2mm]x=-\dfrac{1}{3}\\[2mm]y=-\dfrac{2}{3}\\[2mm]z=\dfrac{5}{3}\end{cases}\end{align*}$
    Donc $K$ a pour coordonnées $\left(-\dfrac{1}{3};-\dfrac{2}{3};\dfrac{5}{3}\right)$.
    $\quad$
  4. a. $\vect{DK}\begin{pmatrix}-\dfrac{10}{3}\\[2mm]-\dfrac{5}{3}\\[2mm]-\dfrac{10}{3}\end{pmatrix}$
    $\begin{align*} DK&=\sqrt{\left(-\dfrac{10}{3}\right)^2+\left(-\dfrac{5}{3}\right)^2+\left(-\dfrac{10}{3}\right)^2} \\
    &=\sqrt{\dfrac{100}{9}+\dfrac{25}{9}+\dfrac{100}{9}} \\
    &=\sqrt{25} \\
    &=5 \text{ cm}\end{align*}$
    $\quad$
    b. Le volume du tétraèdre $DEFG$ est :
    $\begin{align*} V&=\dfrac{1}{3}\times \mathscr{A}\times DK \\
    &=\dfrac{1}{3}\times 12\times 5 \\
    &=20\text{ cm}^3\end{align*}$
    $\quad$

 

Ex 4

Exercice 4

  1. Pour tout réel $x>1$ on a $f(x)=0,05-\dfrac{\ln(x)}{x}\times \dfrac{x}{x-1}$.
    D’après la limite des termes de plus haut degré $\lim\limits_{x\to +\infty} \dfrac{x}{x-1}=\lim\limits_{x\to +\infty} \dfrac{x}{x}=1$.
    Par croissances comparées $\lim\limits_{x\to +\infty} \dfrac{\ln(x)}{x}=0$.
    Ainsi $\lim\limits_{x\to +\infty} f(x)=0,05$.
    Réponse b
    $\quad$
  2. La fonction $h$ est continue sur l’intervalle $[-2;4]$ et donc également sur l’intervalle $[1;3]$.
    $h(1)=4>0$ et $h(3)=-1<0$.
    D’après le théorème des valeurs intermédiaires, l’équation $h(x)=0$ admet au moins une solution sur l’intervalle $[1;3]$.
    Réponse c
    $\quad$
  3. $\lim\limits_{n\to +\infty} u_n=+\infty$. Il existe donc un entier naturel $N$ tel que, pour tout $n\pg N$, on ait $u_n\pg 1$.
    Par conséquent, pour tout $n\pg N$ : $0\pp \dfrac{1}{u_n} \pp 1$ et $0\pp \dfrac{v_n}{u_n}\pp v_n$.
    $\lim\limits_{n\to +\infty} v_n=0$.
    D’après le théorème des gendarmes, $\lim\limits_{n\to +\infty} \dfrac{v_n}{u_n}=0$.
    Réponse b
    $\quad$
  4. On considère la variable aléatoire $X$ égale au gain algébrique du joueur.
    $P(X=8)=\dfrac{1}{6}$ (s’il obtient $1$)
    $P(X=-1)=\dfrac{1}{2}$ (s’il obtient un nombre pair)
    $P(X=-4)=\dfrac{1}{3}$ (sinon)
    L’espérance de $X$ est :
    $\begin{align*} E(X)&=8\times \dfrac{1}{6}-1\times \dfrac{1}{2}-4\times \dfrac{1}{3} \\
    &=-\dfrac{1}{2}\end{align*}$
    Réponse d
    $\quad$
  5. $\quad$
    $\begin{align*} P(X=0)=\dfrac{1}{125}&\ssi (1-p)^3=\dfrac{1}{125} \\
    &\ssi 1-p=\dfrac{1}{5} \\
    &\ssi p=\dfrac{4}{5}\end{align*}$
    Réponse c
    $\quad$

 

 

Énoncé

La qualité de la rédaction, la clarté et la précision des raisonnements seront prises en compte dans l’appréciation de la copie. Les traces de recherche, même incomplètes ou infructueuses, seront valorisées.

Exercice 1     5 points

Partie A

Le plan est muni d’un repère orthogonal.

On considère une fonction $f$ définie et dérivable sur $\R$. On note $f’$ sa fonction dérivée. On donne ci-dessous la courbe représentative de la fonction dérivée $f’$.

Dans cette partie, les résultats seront obtenus par lecture graphique de la courbe représentative de la fonction dérivée $f’$. Aucune justification n’est demandée.

  1. Donner le sens de variation de la fonction $f$ sur $\R$. On utilisera des valeurs approchées si besoin.
    $\quad$
  2. Donner les intervalles sur lesquels la fonction $f$ semble être convexe.
    $\quad$

Partie B

On admet que la fonction $f$ de la partie A est définie sur $\R$ par $f(x)=\left(x^{2}-5 x + 6\right) \e^{x}$.
On note $\mathcal{C}$ la courbe représentative de la fonction $f$ dans un repère.

  1. a. Déterminer la limite de la fonction $f$ en $+\infty$.
    $\quad$
    b. Déterminer la limite de la fonction $f$ en $-\infty$.
    $\quad$
  2. Montrer que, pour tout réel $x$, on a $f'(x)=\left(x^{2}-3 x+1\right) \e^{x}$.
    $\quad$
  3. En déduire le sens de variation de la fonction $f$.
    $\quad$
  4. Déterminer l’équation réduite de la tangente $(\mathcal{T})$ à la courbe $\mathcal{C}$ au point d’abscisse $0$.
    $\quad$

On admet que la fonction $f$ est deux fois dérivable sur $\R$. On note $f”$ la fonction dérivée seconde de la fonction $f$. On admet que, pour tout réel $x$, on a $f”(x) = (x+1)(x- 2) \e^{x}$.

  1. a. Étudier la convexité de la fonction $f$ sur $\R$.
    $\quad$
    b. Montrer que, pour tout $x$ appartenant à l’intervalle $[-1~;~2]$, on a $f(x) \pp x + 6$.
    $\quad$

$\quad$

Exercice 2     5 points

On étudie un groupe de $3~000$ sportifs qui pratiquent soit l’athlétisme dans le club A, soit le basketball dans le club B.

En 2023, le club A compte $1~700$ membres et le club B en compte $1~300$.

On décide de modéliser le nombre de membres du club A et du club B respectivement par deux suites $\left(a_{n}\right)$ et $\left(b_{n}\right)$, où $n$ désigne le rang de l’année à partir de 2023.
L’année 2023 correspond au rang $0$. On a alors $a_{0}= 1~700$ et $b_{0} = 1~300$.

Pour notre étude, on fait les hypothèses suivantes :

  • durant l’étude, aucun sportif ne quitte le groupe ;
  • chaque année, $15\%$ des sportifs du club A quittent le club et adhèrent au club B ;
  • chaque année, $10\%$ des sportifs du club B quittent ce club et adhèrent au club $A$.
  1. Calculer les nombres de membres de chaque club en 2024.
    $\quad$
  2. Pour tout entier naturel $n$, déterminer une relation liant $a_{n}$ et $b_{n}$.
    $\quad$
  3. Montrer que la suite $\left(a_{n}\right)$ vérifie la relation suivante pour tout entier naturel $n$, on a : $a_{n+1}= 0,75 a_{n} + 300$
    $\quad$
  4. a. Démontrer par récurrence que pour tout entier naturel $n$, on a : $$1~200 \pp a_{n+1} \pp a_{n} \pp 1~700$$
    $\quad$
    b. En déduire que la suite $\left(a_{n}\right)$ converge.
    $\quad$
  5. Soit $\left(v_{n}\right)$ la suite définie pour tout entier naturel $n$ par $v_{n}=a_{n}- 1~200$.
    a. Démontrer que la suite $\left(v_{n}\right)$ est géométrique.
    $\quad$
    b. Exprimer $v_{n}$ en fonction de $n$.
    $\quad$
    c. En déduire que pour tout entier naturel $n, a_{n}= 500 \times 0,75^{n}+ 1~200$.
    $\quad$
  6. a. Déterminer la limite de la suite $\left(a_{n}\right)$.
    $\quad$
    b. Interpréter le résultat de la question précédente dans le contexte de l’exercice.
    $\quad$
  7. a. Recopier et compléter le programme Python ci-dessous afin qu’il renvoie la plus petite valeur de $n$ à partir de laquelle le nombre de membres du club A est strictement inférieur à $1~280$.
    $$\begin{array}{|l|}
    \hline
    \textbf{def }\text{seuil() :}\\
    \quad\text{n = 0}\\
    \quad \text{A = 1700}\\
    \quad \textbf{while} \text{ … :}\\
    \qquad \text{n = n + 1} \phantom{123456789}\\
    \qquad \text{A = …}\\
    \quad \textbf{return}\text{ …}\\
    \hline
    \end{array}$$
    $\quad$
    b. Déterminer la valeur renvoyée lorsqu’on appelle la fonction $\text{seuil}$.
    $\quad$

$\quad$

Exercice 3     5 points

Dans l’espace muni d’un repère orthonormé d’unité $1$ cm, on considère les points $$D(3;1;5) \qquad E(3;-2;-1) \qquad F(-1;2;1) \qquad G(3;2;-3)$$

  1. a. Déterminer les coordonnées des vecteurs $\vect{EF}$ et $\vect{FG}$.
    $\quad$
    b. Justifier que les points $E$, $F$ et $G$ ne sont pas alignés.
    $\quad$
  2. a. Déterminer une représentation paramétrique de la droite $(FG)$.
    $\quad$
    b. On appelle $H$ le point de coordonnées $(2;2;-2)$.
    Vérifier que $H$ est le projeté orthogonal de $E$ sur la droite $(FG)$ .
    $\quad$
    c. Montrer que l’aire du triangle $EFG$ est égale à  $12$ cm$^{2}$.
    $\quad$
  3. a. Démontrer que le vecteur $\vec{n}\begin{pmatrix}2 \\ 1 \\ 2\end{pmatrix}$ est un vecteur normal au plan $(EFG)$.
    $\quad$
    b. Déterminer une équation cartésienne du plan $(EFG)$.
    $\quad$
    c. Déterminer une représentation paramétrique de la droite $(d)$ passant par le point $D$ et orthogonale au plan $(EFG)$.
    $\quad$
    d. On note $K$ le projeté orthogonal du point $D$ sur le plan $(EFG)$.
    À l’aide des questions précédentes, calculer les coordonnées du point $K$.
    $\quad$
  4. a. Vérifier que la distance $DK$ est égale à $5$ cm.
    $\quad$
    b. En déduire le volume du tétraèdre $DEFG$.
    $\quad$

$\quad$

Exercice 4     5 points

Cet exercice est un questionnaire à choix multiple. Pour chaque question, une seule des quatre réponses proposées est exacte. Le candidat indiquera sur sa copie le numéro de la question et la réponse choisie. Aucune justification n’est demandée.
Une réponse fausse, une réponse multiple ou l’absence de réponse à une question ne rapporte ni n’enlève de point. Les cinq questions sont indépendantes.

  1. On considère la fonction $f$ définie sur l’intervalle $]1~;+\infty[$ par $f(x)= 0,05-\dfrac{\ln x}{x- 1}$.
    La limite de la fonction $f$ en $+\infty$ est égale à :
    a. $+\infty$
    b. $0,05$
    c. $-\infty$
    d. $0$
    $\quad$
  2. On considère une fonction $h$ continue sur l’intervalle $[-2 ; 4]$ telle que : $$h(-1)=0, \qquad h(1) = 4, \qquad h(3) = -1$$
    On peut affirmer que :
    a. la fonction $h$ est croissante sur l’intervalle $[-1~;~1]$.
    b. la fonction $h$ est positive sur l’intervalle $[-1~;~1]$.
    c. il existe au moins un nombre réel $a$ dans l’intervalle $[1;3]$ tel que $h(a) = 1$.
    d. l’équation $h(x)=1$ admet exactement deux solutions dans l’intervalle $[-2;4]$.
    $\quad$
  3. On considère deux suites $\left(u_{n}\right)$ et $\left(v_{n}\right)$ à termes strictement positifs telles que $\lim\limits_{n \to+\infty} u_{n}=+\infty$ et $\left(v_{n}\right)$ converge vers $0$.
    On peut affirmer que :
    a. la suite $\left(\dfrac{1}{v_{n}}\right)$ converge.
    b. la suite $\left(\dfrac{v_{n}}{u_{n}}\right)$ converge.
    c. la suite $\left(u_{n}\right)$ est croissante.
    d. $\lim\limits_{n \to +\infty}\left(-u_{n}\right)^{n}=-\infty$
    $\quad$
  4. Pour participer à un jeu, un joueur doit payer $4$ €.
    Il lance ensuite un dé équilibré à six faces :
    $\bullet$ s’il obtient $1$, il remporte $12$ €;
    $\bullet$ s’il obtient un nombre pair, il remporte $3$ €;
    $\bullet$ sinon, il ne remporte rien.
    En moyenne, le joueur :
    a. gagne $3,50$ €.
    b. perd $3$ €.
    c. perd $1,50$ €.
    d. perd $0,50$ €.
    $\quad$
  5. On considère la variable aléatoire $X$ suivant la loi binomiale $\mathcal{B}(3~;~p)$.
    On sait que $P(X = 0) = \dfrac{1}{125}$. On peut affirmer que :
    a. $p = \dfrac{1}{5}$
    b. $P(X = 1) =\dfrac{124}{125}$
    c. $p = \dfrac{4}{5}$
    d. $P(X= 1) =\dfrac{4}{5}$
    $\quad$

$\quad$